You are on page 1of 60

Hematology & Oncology – Part 2 14Mar2009

Hematology/Oncology #1 – Immunology: Cancer


1) Which of the following cases occurring during B cell development would be lethal for
the lymphocyte?
a) Translocation of an oncogene resulting in permanent activation of the oncogene
b) Translocation of an oncogene with an immunoglobulin gene promoter
c) Chromosomal breaks during class switching and somatic hypermutation
d) Joining of chromosomal segments from different chromosomes
e) Repair of chromosomal breaks during immune-receptor gene recombination
2.1) What is the role of the oncogene c-myc, which is commonly translocated to the
immunoglobulin heavy chain on chromosome 8?
a) Stimulates cell apoptosis
b) Stimulates lymphocyte proliferation
c) Permanently prevents cell division
d) Protects against cell apoptosis
e) Destroys cancer cells
2.2) What is the role of the oncogene Bcl-2, which is commonly translocated to
chromosomes 14 and 18?
a) Stimulates cell apoptosis
b) Stimulates lymphocyte proliferation
c) Permanently prevents cell division
d) Protects against cell apoptosis
e) Destroys cancer cells
3.1) Which of the following viruses is associated with Kaposi sarcoma, which is
specifically seen in immunocompromised patients (e.g. AIDS)?
a) Human Herpes Virus 3 (VZV)
b) Human Herpes Virus 4 (EBV)
c) Human Herpes Virus 5 (CMV)
d) Human Herpes Virus 8 (HHV-8)
e) Human T lymphocyte virus 1 (HTLV1)
3.2) Which of the following is a retrovirus that encodes Tax protein, leading to a rare
malignancy?
a) Human Herpes Virus 3 (VZV)
b) Human Herpes Virus 4 (EBV)
c) Human Herpes Virus 5 (CMV)
d) Human Herpes Virus 8 (HHV-8)
e) Human T lymphocyte virus 1 (HTLV1)
3.3) Burkitt lymphoma is a consequence of polyclonal activation of B cells by both
malaria and which of the following?
a) Human Herpes Virus 3 (VZV)
b) Human Herpes Virus 4 (EBV)
c) Human Herpes Virus 5 (CMV)
d) Human Herpes Virus 8 (HHV-8)
e) Human T lymphocyte virus 1 (HTLV1)
4.1) Monoclonal gammopathy of undetermined significance (MGUS) are due to
monoclonal immunoglobulins produced in response to some infections. They may also be
present in what healthy population?

DO NOT DISTRIBUTE -1-


Hematology & Oncology – Part 2 14Mar2009

a) Infants
b) Children
c) Hospital workers
d) Adults
e) Elderly
4.2) How is lymphoma usually diagnosed?
a) Morphology in blood or bone marrow
b) Presence of T cell receptor genes
c) Abnormal monoclonal cells
d) Lymph node biopsy
e) Flow cytometry
4.3) How is chronic lymphocytic leukemia (CLL) distinguished?
a) Morphology in blood or bone marrow
b) Presence T cell receptor genes
c) Abnormal monoclonal cells
d) Lymph node biopsy
e) Flow cytometry
4.4) How is acute lymphocytic leukemia (ALL) usually diagnosed?
a) Morphology in blood or bone marrow
b) Presence T cell receptor genes
c) Abnormal monoclonal cells
d) Lymph node biopsy
e) Flow cytometry
4.5) How is lymphoid malignancy diagnosed?
a) Morphology in blood or bone marrow
b) Presence T cell receptor genes
c) Abnormal monoclonal cells
d) Lymph node biopsy
e) Flow cytometry
5) What type of tumor antigen is involved in tyrosinase-secreting melanocytes?
a) Developmental proteins
b) Lineage-specific proteins
c) Viral proteins
d) Proteins produced through translocations
e) None of the above
6) Which of the following tend to be the most immunogenic as they are not present in a
normal individual?
a) Developmental proteins
b) Lineage-specific proteins
c) Viral proteins
d) Proteins produced through translocations
e) None of the above
7) What immune cells are required to eradicate solid tumors?
a) Neutrophils
b) Lymphocyte B cells
c) Lymphocyte T cells

DO NOT DISTRIBUTE -2-


Hematology & Oncology – Part 2 14Mar2009

d) Lymphocyte NK cells
e) Macrophages
8) Tumors are able to evade the adaptive immune system by losing the ability to present
antigen to T cells or by decreasing the expression of major histocompatibility complex
(MHC). Cells expressing low levels of MHC may be targeted for destruction by what
type of immune cell, acting as a backup system?
a) Neutrophils
b) Lymphocyte B cells
c) Lymphocyte T cells
d) Lymphocyte NK cells
e) Macrophages
9) Infusions of anti-CD20 monoclonal antibodies (passive immunotherapy) can reduce or
cure up to 50% of which of the following?
a) B cell lymphoma
b) T cell lymphoma
c) NK cell lymphoma
d) Lymphomatoid papulosis
e) Sézary disease
10) An infusion of which of the following would activate T cells directly (active
immunotherapy), but has side effects including capillary leak syndrome?
a) Cytokine IL-2
b) Cytokine interferons (IFN)
c) Cytokine IL-10
d) Cytokines IL-1 and IL-18
e) Lymphokine-activated killer (LAK) cells

Hematology/Oncology #2 – Pathology
1.1) What is the most common type of non-Hodgkin lymphoma in the United States,
which have a characteristic t(14;18) and cells that are awry or with effacement?
a) Acute lymphoblastic leukemia (ALL)
b) Chronic lymphocytic leukemia (CLL)
c) Small lymphocytic leukemia (SLL)
d) Follicular lymphoma
e) Diffuse large B-cell lymphoma
1.2) What is the type of the most common primary tumor arising in bone of adults?
a) Burkitt lymphoma
b) Mantle cell lymphoma (MCL)
c) Marginal zone lymphoma (MALToma)
d) Plasma cell neoplasms (e.g. multiple myeloma)
e) Lymphoplasmacytic lymphoma (Waldenström macroglobulinemia)
2) Which of the following type for Hodgkin Disease is the most common?
a) Nodular sclerosis
b) Mixed cellularity
c) Lymphocyte-rich
d) Lymphocyte-depletion
e) Lymphocyte predominance (NLPHL)

DO NOT DISTRIBUTE -3-


Hematology & Oncology – Part 2 14Mar2009

3) A patient is diagnosed with Hodgkin lymphoma that affects lymph node regions on
both sides of the diaphragm. What stage is this?
a) Stage I
b) Stage II
c) Stage III
d) Stage IV
4.1) Which of the following acute myelogenous leukemia (AML) classifications in the
French-American-British (FAB) system is the most common and is most associated with
a t(8;21) translocation?
a) M0: Minimally differentiated AML
b) M1: AML without differentiation
c) M2: AML with maturation
d) M3: Acute promyelocytic leukemia
e) M4: Acute myelomonocytic leukemia
f) M5: Acute monocytic leukemia
g) M6: Acute erythroleukemia
h) M7: Acute megakaryocytic leukemia
4.2) Which of the following has a high incidence of disseminated intravascular
coagulation (DIC) and a strong association with t(15;17)?
a) M0: Minimally differentiated AML
b) M1: AML without differentiation
c) M2: AML with maturation
d) M3: Acute promyelocytic leukemia
e) M4: Acute myelomonocytic leukemia
4.3) Which of the following has monoblasts with nonspecific esterases and a subset
associated with inv(16)?
a) M0: Minimally differentiated AML
b) M1: AML without differentiation
c) M2: AML with maturation
d) M3: Acute promyelocytic leukemia
e) M4: Acute myelomonocytic leukemia
4.4) Which of the following has specific antibodies against GP IIb/IIIa or vWF as well as
myelofibrosis or increased marrow reticulin?
a) M7: Acute megakaryocytic leukemia
b) M6: Acute erythroleukemia
c) M5: Acute monocytic leukemia
d) M4: Acute myelomonocytic leukemia
e) M3: Acute promyelocytic leukemia
5) Chronic myeloproliferative syndromes are neoplasms of multipotent progenitor cells,
which give rise to many mature cells and generally do not affect terminal differentiation.
Which syndrome is associated with splenomegaly (secondary hematopoiesis)?
a) Chronic myelogenous leukemia
b) Polycythemia vera
c) Essential thrombocytosis
d) Primary myelofibrosis
e) All of the above

DO NOT DISTRIBUTE -4-


Hematology & Oncology – Part 2 14Mar2009

6.1) A 60-year-old man presents with possible lymphoma. Lab testing reveals
hypogammaglobulinemia. Blood smear is shown, noting parachute smudge cells. Which
of the following is most likely?
a) Acute lymphocytic leukemia (ALL)
b) Chronic lymphocytic leukemia (CLL)
c) Small lymphocytic leukemia (SLL)
d) Hairy cell leukemia (HCL)
e) Diffuse large B-cell lymphoma
6.2) Which of the following markers would NOT be present in CLL?
a) CD5
b) CD10
c) CD19
d) CD20
e) CD23
6.3) What is the approximate median survival time for patients with CLL?
a) 1-year
b) 3-years
c) 5-years
d) 10-years
e) 20-years
7) A middle-aged Caucasian male presents for a routine physical exam. History reveals
several short-term respiratory infections. Exam reveals massive splenomegaly. Lab test
show mild pancytopenia. Bone marrow reveals abnormal cells (shown). Which of the
following is most likely?
a) Acute lymphocytic leukemia (ALL)
b) Chronic lymphocytic leukemia (CLL)
c) Small lymphocytic leukemia (SLL)
d) Hairy cell leukemia (HCL)
e) Diffuse large B-cell lymphoma
8.1) Which of the following is associated with Bence-Jones proteins, a monoclonal
immunoglobulin spike, “stacked coins” or “stacked poker chips” (rouleaux) formations of
red blood cells (shown), and amyloidosis?
a) Burkitt lymphoma
b) Hairy cell leukemia (HCL)
c) Multiple myeloma
d) Plasmacytoma (solitary myeloma)
e) Waldenström macroglobulinemia
8.2) Plasmacytoma affects the soft tissue and thus, unlike multiple myeloma, would not
have bone changes and the resultant:
a) Hypercalcemia
b) Hypocalcemia
c) Hypermagnesemia
d) Hypomagnesemia
e) Hypernatremia
8.3) Increased amounts of what cytokine are associated with poorer multiple myeloma
prognosis, as myeloma cells are dependent on that cytokine?

DO NOT DISTRIBUTE -5-


Hematology & Oncology – Part 2 14Mar2009

a) Interleukin-2 (IL-2)
b) Interleukin-4 (IL-4)
c) Interleukin-6 (IL-6)
d) Interleukin-8 (IL-8)
e) Interleukin-10 (IL-10)
9) A 60-year-old presents with fatigue, weight loss, vision disturbances, and chronic
oozing blood from the gums. Blood electrophoresis shows an M spike. Neoplastic cells
are found in many places including the spleen and lymph nodes. Electrolyte labs show no
increase in calcium. Bone aspiration shows Russell bodies (shown) and Dutcher bodies
(shown). Hyperviscosity syndrome is suspected. Which of the following is most likely?
a) Burkitt lymphoma
b) Hairy cell leukemia (HCL)
c) Multiple myeloma
d) Plasmacytoma (solitary myeloma)
e) Waldenström macroglobulinemia
10) A 4-year-old child presents with 30% blast cells, anemia, and petechiae. Which of the
following is most likely?
a) Acute lymphoblastic leukemia (ALL), B-cell
b) Chronic lymphocytic leukemia (CLL), B-cell
c) Acute lymphoblastic leukemia (ALL), T-cell
d) Chronic lymphocytic leukemia (CLL), T-cell
11) A teenager presents with 30% blast cells, anemia, and mediastinal thoracic pain.
Which of the following is most likely?
a) Acute lymphoblastic leukemia (ALL), B-cell
b) Chronic lymphocytic leukemia (CLL), B-cell
c) Acute lymphoblastic leukemia (ALL), T-cell
d) Chronic lymphocytic leukemia (CLL), T-cell
12) Which of the following is seen in adults, involves basophilic leukocytosis, and is
associated with Philadelphia chromosome t(9;22)?
a) Acute lymphoblastic leukemia (ALL)
b) Chronic lymphocytic leukemia (CLL)
c) Acute myelogenous leukemia (AML)
d) Chronic myelogenous leukemia (CML)
e) Hairy cell leukemia
13) An elderly patient presents with persistent lymphocytosis. Testing reveals no blast
cells and possible Richter syndrome. Which of the following is most likely?
a) Acute lymphoblastic leukemia (ALL)
b) Chronic lymphocytic leukemia (CLL)
c) Acute myelogenous leukemia (AML)
d) Chronic myelogenous leukemia (CML)
e) Hairy cell leukemia
14) An elderly patient presents with complains of early satiety. Examination reveals
splenomegaly. Testing reveals the patient is positive for tartate resistant acid phosphatase
(TRAP+). Which of the following is most likely?
a) Acute lymphoblastic leukemia (ALL)
b) Chronic lymphocytic leukemia (CLL)

DO NOT DISTRIBUTE -6-


Hematology & Oncology – Part 2 14Mar2009

c) Acute myelogenous leukemia (AML)


d) Chronic myelogenous leukemia (CML)
e) Hairy cell leukemia
15) Which of the following signs or symptoms of Hodgkin lymphoma is most common in
women?
a) Reed-Sternberg cells
b) Severe pain
c) Splenomegaly
d) Nodular sclerosis
e) Night sweats
16) What is the Cotswolds Staging Classification for Hodgkin lymphoma that affects
only the spleen?
a) Stage I
b) Stage II
c) Stage III
d) Stage IV
17) An American man presents with stomach pain. Testing reveals lesions in the
abdomen and gastric lymphoma. The clinician suspects that if genetic testing were
performed, a t(8;14) translocation would be found. Which of the following is most likely?
a) Burkitt lymphoma
b) Mycosis fungoides
c) Follicular lymphoma
d) Chronic myelogenous leukemia
e) Waldenström macroglobulinemia
18) Which of the following is NOT associated with multiple myeloma?
a) Punched out lesions (e.g. skull holes)
b) Lytic bone lesions (e.g. back pain)
c) Hypocalcemia
d) Bence-Jones proteins in urine
e) IgG M protein (paraprotein)
19) Burkitt lymphoma commonly affects what area of the body in patients from Africa?
a) Abdomen
b) Spleen
c) Jaw
d) Groin
e) Liver
20) A patient presents with rash-like lesions, skin tumors, and pruritis (itching). Biopsy
reveals the tumors are T-cell in origin. A blood smear is shown, displaying Sézary cells.
Which of the following is most likely?
a) Burkitt lymphoma
b) Mycosis fungoides
c) Follicular lymphoma
d) Chronic myelogenous leukemia
e) Waldenström macroglobulinemia
21) Which of the following is associated with increased IgM when cold (cold globulin),
hypercoagulable state, and thrombotic disease with obstruction to blood flow?

DO NOT DISTRIBUTE -7-


Hematology & Oncology – Part 2 14Mar2009

a) Burkitt lymphoma
b) Mycosis fungoides
c) Follicular lymphoma
d) Chronic myelogenous leukemia
e) Waldenström macroglobulinemia

Hematology/Oncology #3 – Pharmacology: Immunopharmacology


1) All of the following drugs block T cell and B cell proliferation. Which of the following
also blocks antigen processing and interleukin production?
a) Prednisone
b) Azathioprine
c) Methotrexate
d) Cyclophosphamide
e) Mycophenolic acid
2) What is the mechanism of action for muromonab (OKT3)?
a) Inhibition of antigen processing
b) Inhibition of interleukin production from antigen processing cell
c) Inhibition of CD3 on helper T cell
d) Inhibition of interleukin-2 production from helper T cell
e) Inhibition of T cell and B cell proliferation
3) What is the mechanism of action for tacrolimus (FK-506, Fujimycin, Prograf) and
cyclosporine (Sandimmune, Neorall)?
a) Inhibition of antigen processing
b) Inhibition of interleukin production from antigen processing cell
c) Inhibition of CD3 on helper T cell
d) Inhibition of interleukin-2 production from helper T cell
e) Inhibition of T cell and B cell proliferation
4) Which of the following drugs primarily targets cyclophiline and is usually given with
prednisone concurrently?
a) Tacrolimus
b) Cyclosporine
c) Cyclophosphamide
d) Azathioprine
e) Methotrexate
f) Muromonab
g) Prednisone
5) Cyclosporine is bound in the blood, mostly to red blood cells. Along with infection,
what is the most serious side-effect of this drug, which may occur in as many as 75% of
patients?
a) Hirsutism
b) Hepatotoxicity
c) Cardiotoxicity
d) Nephrotoxicity
e) Malignant lymphoma
6) When cyclosporine is combined with a P-450 inducer (phenytoin, phenobarbital,
rifampin, cotrimazole), what is the result?

DO NOT DISTRIBUTE -8-


Hematology & Oncology – Part 2 14Mar2009

a) Cyclosporine metabolism is increased


b) Cyclosporine metabolism is decreased
c) The P-450 inducer is metabolized faster
d) The P-450 inducer is metabolized slower
7) Which of the following is NOT a P-450 inhibitor?
a) Ketoconazole
b) Azithromycin
c) Erythromycin
d) Amphotericin B
e) Grapefruit juice
8) Which of the following inhibits calcineurin?
a) Tacrolimus
b) Cyclosporine
c) Cyclophosphamide
d) Azathioprine
e) Methotrexate
f) Muromonab
g) Prednisone
9) What is the major toxicity seen with tacrolimus (33-40%)?
a) Anaphalaxis
b) Hepatotoxicity
c) Nephrotoxicity
d) Serious infection
e) Malignant lymphoma
10) Which of the following would occur if a patient is on tacrolimus, drinks a liter of
grapefruit juice per day, and take the antifungal fluconazole?
a) Fluconazole metabolism will increase (drug levels decrease)
b) Fluconazole metabolism will decrease (drug levels increase)
c) Tacrolimus metabolism will increase (drug levels decrease)
d) Tacrolimus metabolism will decrease (drug levels increase)
11) A patient presents with sudden leg pain, severe vomiting, and low blood pressure.
History reveals the patient had their medications stolen at the airport prior to a cruise
vacation. Addisonian crisis is diagnosed. What drug was the patient likely taking?
a) Didronel
b) Methotrexate
c) Tacrolimus
d) Cyclophosphamide
e) Prednisone
12) Which of the following is NOT a side effect associated with glucocorticosteroids
(e.g. prednisone)?
a) Thinning of the skin
b) Osteoporosis
c) Weight gain
d) Hypotension
e) Infections
13) Which of the following cytotoxic drugs is specific to B and T lymphocytes?

DO NOT DISTRIBUTE -9-


Hematology & Oncology – Part 2 14Mar2009

a) Mycophenolate mofetil (CellCept)


b) Methotrexate (MTX, Folex)
c) Cyclophosphamide (Cytoxan)
d) Azathioprine (Immuran)
14) Which of the following drugs interferes with folate metabolism?
a) Cyclophosphamide (Cytoxan)
b) Azathioprine (Immuran)
c) Mycophenolate mofetil (CellCept)
d) Methotrexate (MTX)
15) What drug’s toxicity is increased by allopurinol (Zyloprim) and is associated with
neoplasms with long-term use?
a) Cyclophosphamide (Cytoxan)
b) Azathioprine (Immuran)
c) Methotrexate (MTX, Folex)
d) Mycophenolate mofetil (CellCept)
16) Which of the following drugs can cause neutropenia, hemorrhagic cystitis of the
bladder, and sterility?
a) Cyclophosphamide (Cytoxan)
b) Azathioprine (Immuran)
c) Mycophenolate mofetil (CellCept)
d) Methotrexate (MTX, Folex)
17) Which of the following is a product that must be converted to its active form, 6-MP,
and is able to cross the placenta?
a) Mycophenolate mofetil (CellCept)
b) Methotrexate (MTX, Folex)
c) Cyclophosphamide (Cytoxan)
d) Azathioprine (Immuran)
18) What is the major toxic concern with methotrexate?
a) Cardiotoxicity
b) Hepatotoxicity
c) Nephrotoxicity
d) Ototocicity
e) Optic neuritis
19) Which of the following drugs acts by inhibiting inosine monophosphate
deydrogenase (IMD), which is the rate-limiting enzyme in the formation of guanosine?
a) Mycophenolate mofetil (CellCept)
b) Methotrexate (MTX, Folex)
c) Cyclophosphamide (Cytoxan)
d) Azathioprine (Immuran)
e) Anakinra (Kinaret, Amagen)
20) Which of the following drugs has decreased absorption when combined with antacids
or cholestyramine?
a) Mycophenolate mofetil (CellCept)
b) Methotrexate (MTX, Folex)
c) Cyclophosphamide (Cytoxan)
d) Azathioprine (Immuran)

DO NOT DISTRIBUTE - 10 -
Hematology & Oncology – Part 2 14Mar2009

e) Anakinra (Kinaret, Amagen)


21) Which of the following drugs is associated with neurophychiatric events and
encephalopathy?
a) Tacrolimus
b) Cyclosporine
c) Cyclophosphamide
d) Azathioprine
e) Methotrexate
f) Muromonab
g) Prednisone
22) Which of the following is an interleukin-1 (IL-1) receptor antagonist that is used for
rheumatoid arthritis in adults who have failed at least one DMARD?
a) Mycophenolate mofetil (CellCept)
b) Methotrexate (MTX, Folex)
c) Cyclophosphamide (Cytoxan)
d) Azathioprine (Immuran)
e) Anakinra (Kinaret, Amagen)
23) A patient presents with petechiae of the legs. Inflammation of the blood vessels
(vasculitis) is diagnosed. Which of the following would NOT be a recommended
treatment option?
a) Plasma exchange
b) Anakinra
c) Azathioprine
d) Cyclophosphamide
e) Steroids
24) Along with plasma exchange and predisolone, which of the following is
recommended in the treatment of Goodpasteur disease?
a) Mycophenolate mofetil (CellCept)
b) Methotrexate (MTX, Folex)
c) Cyclophosphamide (Cytoxan)
d) Azathioprine (Immuran)
e) Anakinra (Kinaret, Amagen)
25) Which of the following is recommended for a patient who has acquired coagulation
factor XIII antibodies?
a) Mycophenolate mofetil (CellCept)
b) Methotrexate (MTX, Folex)
c) Cyclophosphamide (Cytoxan)
d) Azathioprine (Immuran)
e) Anakinra (Kinaret, Amagen)
26) Thymosin serum levels are normally high in which of the following situations?
a) Children
b) Adults
c) Elderly
d) DiGeorge Syndrome
27) Which of the following is FDA approved for hepatitis B and C?
a) Thymosin alpha-1

DO NOT DISTRIBUTE - 11 -
Hematology & Oncology – Part 2 14Mar2009

b) Thymus-related peptides
c) Interferon (IFN) alpha
d) Interferon (IFN) beta
e) Interferon (IFN) gamma
28) Which of the following is FDA approved for relapsing-type multiple sclerosis (MS)?
a) Thymosin alpha-1
b) Thymus-related peptides
c) Interferon (IFN) alpha
d) Interferon (IFN) beta
e) Interferon (IFN) gamma
29) Which of the following is given in combination with fluorouracil for treating post-
surgical colorectal cancer?
a) Roquinimex
b) Levamisole
c) BCG (Bacille Balmette-Guerin)
d) Interleukin-2
e) Interferon (IFN) gamma
30) Which of the following may work by activating macrophages, is an immunization for
tuberculosis, and is used as an immunostimulant in cancer?
a) Roquinimex
b) Levamisole
c) BCG (Bacille Balmette-Guerin)
d) Interleukin-2
e) Interferon (IFN) gamma

Hematology/Oncology #4 – Pharmacology: Cancer Chemotherapy


1) Which of the following would be considered Ames test negative (non-carcinogenic)?
a) Tobacco
b) Azo dyes
c) Nitrous oxide
d) Benzene
e) Aflatoxins
2) Which of the following describes the chemotherapeutic index (CTI)?
a) ED50 for cancer cells / ED50 for normal cells
b) ED50 for normal cells / ED50 for cancer cells
c) LD50 for cancer cells / LD50 for normal cells
d) LD50 for normal cells / LD50 for cancer cells
e) ED50 for cancer cells / LD50 for normal cells
3) Mitosis begins after what phase of the cell cycle?
a) G1
b) G2
c) S (synthesis)
d) R (restriction point)
e) G0 (resting)
4) What phase of the cell cycle do vincristine and vinblastine affect?
a) Synthesis

DO NOT DISTRIBUTE - 12 -
Hematology & Oncology – Part 2 14Mar2009

b) Mitosis
c) G1
d) G2
e) Non-specific
5) What phase of the cell cycle is affected by hydroxyurea and cytosine arabinoside?
a) Synthesis
b) Mitosis
c) G1
d) G2
e) Non-specific
6) What phase of the cell cycle is affected by alkylating agents and antitumor antibiotics?
a) Synthesis
b) Mitosis
c) G1
d) G2
e) Non-specific
7) What phase of the cell cycle is affected by paclitaxel?
a) Synthesis
b) Mitosis
c) G1
d) G2
e) Non-specific
8) What phase of the cell cycle is affected by procarbazine and cis-platinum?
a) Synthesis
b) Mitosis
c) G1
d) G2
e) Non-specific
9) What phase of the cell cycle is affected by methotrexate and 6-mercaptopurine?
a) Synthesis
b) Mitosis
c) G1
d) G2
e) Non-specific
10) Tumor growth characteristically exhibits a sigmoid-shaped curve where doubling
time varies with:
a) Patient age
b) Tumor age
c) Tumor location
d) Tumor size
e) Tumor origin
11) A chemotherapy agent that has a 1-log kill would destroy what percentage of cells?
a) 1%
b) 33%
c) 66%
d) 90%

DO NOT DISTRIBUTE - 13 -
Hematology & Oncology – Part 2 14Mar2009

e) 99%
12) A patient is found to have 10^12 tumor cells in their body (neoplastic cell burden).
An effective drug is given which destroys 99.9% of clonogenic tumor cells, leading to
clinical remission. How many logs of tumor cells are still in the patient’s body?
a) Twelve (10^12)
b) Nine (10^9)
c) Six (10^6)
d) Three (10^3)
e) None
13) Which of the following chemotherapy treatment options would most likely increase
survival but not alleviate symptoms?
a) No treatment
b) Infrequent drug therapy
c) Combination drug therapy
d) Surgery and infrequent drug therapy
e) Surgery and combination drug therapy
14) Along with colon cancer, what form of cancer may exhibit primary resistance against
drug therapy?
a) Hodgkin disease
b) Childhood acute leukemia
c) Non-small cell lung cancer
d) Testicular cancer
e) Lymphoma
15) Which of the following would occur with increased expression of the normal MDR1
gene, which encodes for a cell surface glycoprotein (P-glycoprotein)?
a) Anti-tumor drug influx into tumor cells would increase
b) Anti-tumor drug influx into tumor cells would decreased
c) Anti-tumor drug efflux out of tumor cells would increase
d) Anti-tumor drug efflux out of tumor cells would decreased
16) A patient undergoing chemotherapy develops tumor resistance to anthracyclines.
Which of the following drug could be used to overcome this MDR-1-mediated drug
resistance?
a) Calcium channel blockers
b) Paclitaxel (Taxol)
c) Vinca alkaloids
d) Mitomycine (Mutamycine) or plicamycin (Mithramycin)
e) Etoposide
17) Which of the following cancer treatment options is used for systemic tumors?
a) Radiotherapy
b) Chemotherapy
c) Endocrine therapy
d) Immunotherapy
e) Thermotherapy
18) Along with cytopenia, what is the major side-effect of chemotherapy agents?
a) Sexuality impact
b) Weight loss

DO NOT DISTRIBUTE - 14 -
Hematology & Oncology – Part 2 14Mar2009

c) Alopecia
d) Taste changes
e) Nausea and vomiting
19) Which of the following is NOT an effective drug in the management (prophylaxis) of
nausea seen with abdominal radiotherapy and with many chemotherapy agents?
a) Serotonin-receptor antagonists (ondansetron, granisetron, dolasetron)
b) Anti-dopaminergics (prochlorperazine, fluphenazine)
c) Metoclopramide (Reglan)
d) Lysergic acid diethylamide (LSD)
e) Dronabinol (Marinol, marijuana THC)
20) Which of the following drugs is the most effective available for the management of
nausea and vomiting associated with radiotherapy or chemotherapy?
a) Serotonin-receptor antagonists (ondansetron, granisetron, dolasetron)
b) Anti-dopaminergics (prochlorperazine, fluphenazine)
c) Metoclopramide (Reglan)
d) Lysergic acid diethylamide (LSD)
e) Dronabinol (Marinol, marijuana THC)
21) Anti-dopaminergics act as anti-emetics by depressing the:
a) Cerebral cortex
b) Vestibular apparatus
c) Chemoreceptor trigger zone (CTZ)
d) Area postrema (vomiting center)
e) Nucleus of the Solitary tract
22) A chemotherapy patient presents with extrapyramidal signs (acute dystonic reactions
and Parkinson-like symptoms). They are on an anti-emetic that increases the tone and
amplitude of gastric contractions. What drug are they taking?
a) Ondansetron (Zofran)
b) Prochlorperazine (Compazine)
c) Metoclopramide (Reglan)
d) Dronabinol (Marinol)
e) Dolasetron (Anzemet)
23) Which of the following cytopenias seen in cancer chemotherapy involves high fever
and increased risk of infection, and should be treated with G-CSF and antibiotics?
a) Anemia
b) Leukopenia
c) Neutropenia
d) Thrombocytopenia
e) All of the above
24) Activated Protein C (Drotrecogin-alpha, Xigris) is FDA approved in the treatment of
critically ill patients who have:
a) Intracranial hemorrhage
b) Pancytopenia
c) Malignant cancer
d) Cerebral cancer
e) Sepsis

DO NOT DISTRIBUTE - 15 -
Hematology & Oncology – Part 2 14Mar2009

25) A 50-year-old patient is being treated for Hodkin Disease using the MOPP regimen.
Following three days of treatment, the patient develops severe nausea and vomiting. They
are put on ondansetron (Zofran), but develop severe dermatitis. What drug should they be
switched to?
a) Chlorpromazine (Thorazide)
b) Prochlorperazine (Compazine)
c) Metoclopramide (Reglan)
d) Dronabinol (Marinol)
e) Dolasetron (Anzemet)

Hematology/Oncology #5 – Nutrition
1) A colon carcinoma was detected in a 44-year-old woman. At the same site, an
adenoma had been endoscopically removed 5 years earlier. The same oncogene mutation
was detected in both tissue samples from the carcinoma and from the previously removed
adenoma, but not in healthy tissue from this patient. What is the most probably
explanation for the presence of the same mutation in the two tumor tissues?
a) Lack of regular fruit consumption often causes the same oncogene mutations
b) Continued exposure to the same carcinogen caused this mutation
c) Both adenomas and carcinomas typically harbor this kind of mutation
d) Adenoma and the later carcinoma are derived from the same mutated precursor
cell
e) The patient has a familial cancer syndrome caused by the oncogene mutation
2) Several colon polyps are detected in the colon of a 43-year-old man during
radiological examination for an unrelated illness. Why is cancer more likely to develop at
these sites than at non-affected sites?
a) With oncogene mutations already present in polyp cells, additional mutations
are more likely to generate a cancerous cell
b) Since polyps protrude into the interstitial lumen, they are more exposed to
carcinogens
c) Polyps are more cancer-prone because they reflect an accumulation of
carcinogens
d) As a colon polyp grows, all of its cells tend to break through organ barriers and
become more invasive in nature
e) Mutations in the polyp’s cells accelerate mucosal sloughing and interrupt
normal cell signaling
3) A 52-year-old man had a small cancerous colon polyp endoscopically removed several
years earlier, and has not had any recurrences since then. How will his diet most likely
affect his future cancer risk?
a) Eating fruits and vegetables slows the accumulation of DNA mutations
b) Dietary fiber binds and eliminates mutated DNA
c) Mucosal cells with cancer mutations are more likely to revert with a folate-rich
diet
d) Since he is likely to have cancer mutations already, diet will not affect his risk
e) His risk of cancer metastasis will be greatly reduced if he avoids flavonoid-rich
foods

DO NOT DISTRIBUTE - 16 -
Hematology & Oncology – Part 2 14Mar2009

4) DNA is extracted from colon cancer tissues of a 67-year-old vegetarian. It is found to


contain a different form of the p53 gene than DNA from healthy tissues. Which is the
most probably explanation for this difference?
a) The excess carotene consumed by vegetarians causes increased DNA mutation
rates
b) The p53 variants reflect DNA adduct formation caused by high carotene intake
c) All cancer cells contain a variant form of p53
d) The cancer has grown from one specific cell with a p53 mutation
e) There is no connection, since p53 mutation are inherited traits
5) After hearing about the benefits of antioxidants, a chain-smoking 37-year-old man has
started taking a supplement containing 10 times the RDA of vitamin E. How might a
megadose vitamin E supplement actually do more harm than good?
a) Accumulation of excess vitamin E in low-density lipoproteins impairs receptor-
mediated lipoprotein clearance
b) Since vitamin E is fat-soluble, toxic amounts easily accumulate in the liver
c) Excess vitamin E-free radical complexes are excreted with bile and may
damage the DNA of intestinal mucosa
d) In combination with the frequently poor vitamin C status in smokers, excess
vitamin E may increase free radical generation
e) Cleavage of excess alpha-tocopherol generates retinol which is known to
promote tumor cell growth and increase lung cancer risk in smokers
6) Which is the essential cofactor for the reactivation of vitamin E?
a) Xanthine
b) Catalase
c) Ascorbate
d) Beta-carotene
e) Uric acid
7) A single hydroxyl free radical encounters a cell membrane. What changes in the
membrane are to be expected?
a) The polyunsaturated fatty acids in one triglyceride molecule will become
saturated and destabilize the membrane
b) Hydroxylation of a cholesterol molecule generates a cholesterol superoxide
anion
c) The saturated fatty acid in a cholesterol ester will become unsaturated and
increased membrane fluidity
d) A single linoleic acid residue in the membrane will become saturated which
will perturb membrane structure
e) Several membrane phospholipid molecules will be oxidized
8) Which antioxidant defenses protect against excessive free radical damage in cytosol?
a) Vitamin E
b) Vitamin A
c) Linoleic acid
d) Ribosomes
e) Superoxide dismutase
9) A young man prefers to have his hamburgers charbroiled. In what way does this affect
his cancer risk?

DO NOT DISTRIBUTE - 17 -
Hematology & Oncology – Part 2 14Mar2009

a) Charbroiling causes the formation of highly bioavailable nitrosamines


b) The lower fat content of broiled meat reduces cancer risk
c) DNA adducts are formed in the broiled meat which are carcinogenic
d) If hamburgers are not completely cooked, the raw (“red”) meat increases
cancer risk
e) Compounds from broiled meat attach to human DNA and increase mutation
risk
10) Among people who often eat nitrite-cured foods, those with regular fruit and
vegetable consumption have the lowest nitrosamine exposure. What is the most likely
mechanism?
a) The high fiber content of fruits and vegetables accelerates fecal nitrosamine
excretion
b) Carotenoids in fruits and vegetables are free radical scavengers and thus
inactivate nitrosamines
c) Carotenoids in fruits and vegetables form charge-transfer complexes with
nitrosamines which are rapidly conjugated
d) Ascorbic acid in fresh fruits and vegetables inhibits nitrosamine formation in
the stomach
e) Ascorbic acid in fresh fruits and vegetables promotes nitrosamine reduction
and thus inactivation
11) Which of the following is quantitatively the largest dietary source of polycyclic
aromatic hydrocarbons (PAHs)?
a) Chicken soup
b) Charbroiled pork chop
c) Grilled tomato
d) Cheese-lettuce sandwich
e) Apple pie with ice cream
12) Aflatoxins are food contaminants produced by molds. What is the most likely
mechanism whereby these substances affect cancer risk?
a) They form nitrosamines in the stomach which in turn cause DNA mutation
b) They form DNA adducts in exposed tissues
c) They are inhibitors of phase II enzymes
d) They activate phase I enzymes in adipose tissue
e) They inhibit the DNA repair mechanism
13) An elderly man enjoys eating smoked fish. He is also using an anti-epileptic drug
containing barbiturate, which is known to induce phase I enzymes. What is the likely
interaction between his preferred food and his prescription drug?
a) Conjugation of carcinogens from smoked fish by phase I enzymes increase
their reactivity with DNA
b) Polycyclic aromatic hydrocarbons from smoked fish are oxidized by phase I
enzymes which increases their carcinogenicity
c) Phase I enzymes hydroxylate DNA adducts from smoked fish and thereby
increase their carcinogenicity
d) Phase I enzymes generate nitrosamines and thereby increase carcinogenicity
e) Hydroxylation of polycyclic aromatic hydrocarbons from smoke by phase I
enzymes increases DNA adduct formation

DO NOT DISTRIBUTE - 18 -
Hematology & Oncology – Part 2 14Mar2009

14) A middle-aged woman who has been smoking since her teenage years always has
liked broccoli with cheese and eaten generous amounts several times a week. How might
these eating habits influence cancer risk of a smoker?
a) Regular broccoli consumption increases phase I enzyme activity
b) The high fat content of cheese increases estrogen receptor activity which in
turn increases cancer risk
c) Substances in most vegetables enhance the catabolism of DNA adducts
d) Broccoli promotes the conjugation of carcinogens
e) Because broccoli has no direct effect on inhaled carcinogens, lung cancer risk
is unchanged
15) Phase I enzyme-dependent reactions include which of the following activities?
a) Alkylation of neucleotides
b) Glycosylation of oncogenes
c) Glucuronidation of aflatoxins
d) Hydroxylation of nitrosamines
e) Peroxidation of fatty acids
16) Which of the following activities are most likely enhanced by phase II enzyme
activators?
a) Aflatoxin deconjugation
b) Nitrosamine hydroxylation
c) Fatty acid peroxidation
d) Superoxide anion reduction
e) Benzpyrene glucuronidation
17) Which of the following is part of generally accepted guidelines for the dietary
reduction of cancer risk?
a) Prefer fat-free foods
b) Take a daily fiber supplement
c) Eat only organically grown vegetables
d) Use an antioxidant supplement with both vitamin C and E
e) Limit consumption of salt-cured and smoked foods
18) General recommendations for a cancer-preventive diet should include:
a) Achieve ideal body weight status and avoid soyfoods, meats, and alcohol
b) Do not eat barbecued vegetables and have raw vegetables with each meal
c) Avoid meats and refined sugar, and eat five servings of fruit a day
d) Avoid excess body fat, and eat several servings of green and vegetables a day
e) Avoid dietary polyphenols, nitrosamines, free radicals, and phytoestrogens
19) An anti-carcinogenic compound in citrus fruits is:
a) Magnesium
b) Zinc
c) Calcium
d) Riboflavin
e) Vitamin C
20) It has been proposed that a low omega-3 fatty acid intake increases risk of some
cancers. A study investigated this possible link by measuring adipose tissue composition
(a marker for past omega-3 fatty acid intake) in newly diagnosed patients. What
additional information is needed to make this a useful comparison?

DO NOT DISTRIBUTE - 19 -
Hematology & Oncology – Part 2 14Mar2009

a) Time lapse since diagnosis


b) Information on family cancer risk
c) Adipose tissue composition of patients without cancer
d) Endoscopy results showing the absence of colon cancer
e) Dietary assessment of current omega-3 and omega-6 fatty acid intake
21) A recent study investigated the importance of fat consumption for the development of
colon cancer. Cancer patients and randomly selected people from the same community
were asked to complete a questionnaire about their dietary habits during childhood and
total fat intake was calculated from the responses. What type of study is this?
a) Population survey
b) Ecological study
c) Prospective cohort study
d) Case-control study
e) Clinical trial
22) A case-control study is to investigate the suspected role of low lycopene intake as a
risk factor for prostate cancer. Which is the most appropriate diet assessment tool?
a) Single 24-hour diet recall
b) Repeated recall interviews
c) Week-long diet protocol
d) Interview of family members
e) Diet history questionnaire
23) Data from a prospective multifactorial intervention study were analyzed to identify
foods that help prevent cancer. Unexpectedly, participants who developed cancer had
lower consumption of artichokes than those who remained cancer free. A patient who
read about this finding on the internet wants to know whether the findings are
scientifically sound and whether she should increase her artichoke consumption. What is
the most appropriate advice for this patient?
a) There was not a specific hypothesis, and therefore this finding lacks scientific
validity
b) Since the result was unexpected, it is most likely not valid
c) Since the finding is interesting, it will probably lead to formal testing; artichoke
consumption should not be changed pending follow-up results
d) This association is interesting, and since food cannot do much harm, you
advise increased artichoke consumption until follow-up testing is completed
e) Since the study was carefully conducted, the results are valid and you
recommend increased artichoke consumption
24) A prospective study of pack-a-day smokers assessed customary food intakes and
carried out annual diagnostic tests to detect newly formed lung cancers. After ten years,
fewer of the participants with high fruit and vegetable intakes had new cancer than those
with low consumption. What do you need to look for in the report to assess the validity of
the study?
a) Was fat intake monitored?
b) Were cancer patients matched for sex and age?
c) Was cigarette exposure measured with a biomarker?
d) Was dietary information available for most of the participants?
e) Were appropriate markers used to exclude people with genetic cancer risk?

DO NOT DISTRIBUTE - 20 -
Hematology & Oncology – Part 2 14Mar2009

25) A study was undertaken to investigate the hypothesis that lower vitamin C intake
increases lung cancer risk in 40- to 50-year-old male smokers. Vitamin C intakes and
cancer status were assessed by questionnaire seven years after recruitment. Which
essential information is needed to make this a prospective cohort study?
a) Date on a control group of non-smokers
b) Dietary data from time of recruitment
c) Survival time of cancer patients
d) Intermediate marker of cancer
e) Family history of lung cancer
26) High cabbage consumption is associated with low colon cancer risk in many
populations. Which type of study is most suitable to rapidly test the practical relevance of
this initial observation?
a) Ecological study linking isothiocyanate content of cabbages to cancer incidence
b) Blinded cohort study with groups randomized to use isothiocyanate or placebo
c) Prospective cohort study relating isothiocyanate intake from cabbage to cancer
incidence
d) Controlled clinical trial of isothiocyanate using adenomatous polyps as end
point
e) Case-controlled study of cabbage in patients and matched controls
27) A journal article reports that healthy people consume green leafy vegetables about as
often as patients with newly diagnosed colon cancer. Seven-day dietary records were
used to assess food intakes. You know, however, that another study of cancer patients
found a relationship between consumption of green leafy vegetables and the risk of colon
cancer. This second study had used a questionnaire asking cancer patients and controls
about their customary food intakes ten years earlier. Does the second study contradict the
findings of the first one?
a) Yes, since a 7-day dietary record is more accurate than the food frequency
questionnaire used in the previous study
b) No, since the two diet assessment methods have targeted different time periods
c) The question cannot be answered, since dietary assessment methods are too
unreliable
d) No, since 7-day dietary records cannot assess the intake of individual foods
e) Yes, since people know much better what they currently eat than what they are
ten years earlier
28) Mr. Jones has lung cancer and has completed his chemotherapy; he has been losing
weight. What do you think the FIRST recommendation for dietary treatment should be
for him?
a) Increase his protein and calorie intake with regular foods
b) Supplement with high calorie/high protein commercial supplements
c) Prescribe appetite stimulants
d) Provide him with intravenous feeding or tube-feeding
e) None, since his chemotherapy is complete; eventually he will start eating better
29) For the treatment of disseminated bone and liver metastases, a 48-year-old woman
now is undergoing palliative chemotherapy. Which are relevant criteria that should
influence selection of diet?
a) Anti-carcinogenic properties of foods

DO NOT DISTRIBUTE - 21 -
Hematology & Oncology – Part 2 14Mar2009

b) Individual preferences to overcome loss of appetite


c) Low fat content of meals
d) Concentration of phytoestrogens in food
e) Caloric restriction to limit tumor growth

Hematology/Oncology #6 – Clinical: Malignancies


1) Which of the following is needed for a diagnosis of chronic lymphocytic leukemia
(CLL)?
a) Lymphocytosis > 5000/mcL and involves only B cells
b) Hypogammaglobulinemia and recurrent infections
c) >30% lymphocytes in bone marrow and lymphocytosis > 5000/mcL
d) Recurrent infections, age > 60, and autoimmune hemolytic anemia
e) Monoclonal gammopathy and pure RBC aplasia
2) Which of the following is NOT a predictor of decreased survival in CLL?
a) Mutated IGVH genes
b) Expression of CD38
c) Trisomy 12
d) Atypical lymphocyte morphology
e) Deletion of 23q, 17p, or 11q with loss/mutation to p53 gene
3) Which of the following is NOT characteristic of Richter syndrome?
a) Fevers
b) Good prognosis
c) Splenomegaly
d) Asymmetric adenopathy
e) CLL conversion to diffuse large cell lymphoma
4) Chronic lymphocytic leukemia (CLL) is associated with solid tumors of the lung and:
a) Thyroid cancer
b) Adrenal cancer
c) Brain cancer
d) Skin cancer
e) Liver cancer
5) Which of the following lab findings is NOT diagnostic of CLL?
a) Chromosomal abnormalities
b) Phenotype CD19+, CD20+, CD23+, CD5+
c) Lymphocytosis > 5000 mature cells per mcL
d) “Smudged” cells on peripheral blood smear
e) Neoplastic cells expressing low levels of bcl-2
6) A CLL patient with lymphocytosis, anemia with hemoglobin < 10g/dL, and without
thrombocytopenia would be classified as Stage C according to the International
Workshop Classification. What stage would this patient be under the Rai classification?
a) Stage I
b) Stage II
c) Stage III
d) Stage IV
e) None of the above

DO NOT DISTRIBUTE - 22 -
Hematology & Oncology – Part 2 14Mar2009

7) Prognostic factors requiring chemotherapy include trisomy 12, deletion of 11q and
17p, and Rai stage III or IV. Which of the following would NOT be included in the
standard chemotherapy regimen for those patients?
a) Fludarabine (Fludura)
b) Cyclophosphamide (Cytoxan)
c) Rituximab (Rituxan)
d) Vinblastine (Velbe)
e) All are used initially
8) A peripheral blood smear is shown with tartrate-resistant acid phosphates (positive
TRAP stain). Which of the following is most likely?
a) Acute lymphocytic leukemia (ALL)
b) Chronic lymphocytic leukemia (CLL)
c) Small lymphocytic leukemia (SLL)
d) Hairy cell leukemia (HCL)
e) Hodgkin lymphoma
9) Which of the following is NOT characteristic of hairy cell leukemia (HCL)?
a) Splenomegaly
b) More common in males
c) Cytopenia and infections
d) Dry tap on bone marrow aspiration
e) Biopsy specimens are hypocellular
10) Which of the following infections are seen most commonly in hairy cell leukemia?
a) Toxoplasmosis
b) Bacterial pneumonia and urinary tract infections
c) Atypical mycobacterium
d) Viral infections
e) Histoplasmosis and coccidiodes
11) Which of the following drugs produces complete remission rates in about 85% of
patients with hairy cell leukemia after a 7-day continuous IV infusion?
a) Cladribine (Leustatin)
b) Cyclophosphamide (Cytoxan)
c) Vinblastine (Velbe)
d) Rituximab (Rituxan)
e) Fludarabine (Fludura)
12) A 25-year-old man with a history of mononucleosis presents with low hemoglobin
and low serum albumin. After detecting the presence of Reed-Sternberg “owls-eyes”
cells, Hodgkin lymphoma is diagnosed. Which of the following are the most common
infections that this patient is predisposed to?
a) Histoplasmosis and toxoplasmosis
b) Bacterial pneumonia and urinary tract infections
c) Herpes VZV and CMV
d) Herpes EBV and KSHV
e) Legionella and mycoplasma pneumonia
13) An elderly man presents with pruritis, cytopenia, and low
serum albumin. A peripheral blood smear is shown. Which of the
following is most likely?

DO NOT DISTRIBUTE - 23 -
Hematology & Oncology – Part 2 14Mar2009

a) Acute lymphocytic leukemia (ALL)


b) Chronic lymphocytic leukemia (CLL)
c) Small lymphocytic leukemia (SLL)
d) Hairy cell leukemia (HCL)
e) Hodgkin lymphoma
14) Which of the following is NOT a method of staging for Hodgkin lymphoma?
a) Pelvic CT
b) PET scan
c) Abdominal CT
d) MRI scan
e) Chest CT
15) Which of the following describes the reproductive recommendations for patients with
Hodgkin Disease (HD) who undergo treatment?
a) Men should store sperm and women should avoid pregnancy for two years
b) Men should avoid reproduction efforts due to near 100% miscarriage rate
c) Women should avoid reproduction efforts for five years after treatment
d) Men should store sperm and women have no restrictions
e) Both men and women have no reproductive recommendations
16) Which of the following is NOT a Hodgkin disease emergency that is indicated for the
use of radiotherapy?
a) Pericardial tamponade
b) Airway obstruction
c) Superior vena cava syndrome
d) Deep vein thrombosis
e) Epidural spinal cord compression
17.1) Which of the following is considered a B symptom using the Cotswolds Staging
Classification for Hodgkin Disease?
a) Severe nausea, vomiting, and dry heaves
b) Drenching night sweats and weight loss
c) Night terrors and cravings for high fat foods
d) Widening of the mediastinum by more than 1/3
e) Involvement of a single extranodal site that is contiguous or proximal to the
known nodal site
17.2) Which of the following is considered an X symptom (bulky disease) using the
Cotswolds Staging Classification for Hodgkin Disease?
a) Severe nausea, vomiting, and dry heaves
b) Drenching night sweats and weight loss
c) Night terrors and cravings for high fat foods
d) Widening of the mediastinum by more than 1/3
e) Involvement of a single extranodal site that is contiguous or proximal to the
known nodal site
18) Which of the following Cotswolds Staging Classifications is defined as involvement
of lymph node regions on both sides of the diaphragm with or without involvement of
splenic, hilar, celiac, or portal nodes?
a) Stage I
b) Stage II

DO NOT DISTRIBUTE - 24 -
Hematology & Oncology – Part 2 14Mar2009

c) Stage III-1
d) Stage III-2
e) Stage IV
19) What is currently the initial treatment of choice for Hodgkin lymphoma?
a) ABVD (Adriamycin, bleomycin, vinblastine, dacarbazine)
b) BEACOPP (bleomycin, etoposide, Adriamycin, cyclophosphamide, Oncovin,
procarbazine, prednisone)
c) MOPP (Mechlorethamine, Oncovin, procarbazine, prednisone)
d) ChlVPP/EVA (chlorambucil, vincristine, procarbazine, prednisone, etoposide,
vinblastine, Adriamycin)
20) Which of the following chemotherapy options is the preferred treatment for advanced
stages of Hodgkin lymphoma as it results in improved disease-resurvival and overall
survival?
a) ABVD (Adriamycin, bleomycin, vinblastine, dacarbazine)
b) BEACOPP (bleomycin, etoposide, Adriamycin, cyclophosphamide, Oncovin,
procarbazine, prednisone)
c) MOPP (Mechlorethamine, Oncovin, procarbazine, prednisone)
d) ChlVPP/EVA (chlorambucil, vincristine, procarbazine, prednisone, etoposide,
vinblastine, Adriamycin)
e) Stanford V (doxorubicin, mechlorethamine, bleomycin, vinblastine, vincristine,
etoposide, prednisone)
21) Which of the following is the recommended treatment for Hodgkin disease patients
who relapse after initial chemotherapy?
a) Radiation therapy
b) ABVD therapy
c) MOPP therapy
d) BEACOPP therapy
e) Stem cell or bone marrow transplant
22) Which of the following is NOT a complication of Hodgkin disease?
a) Alopecia
b) Infertility
c) Amenorrhea
d) Pneumococcal sepsis
e) Hypothyroidism and thyroid carcinoma
23) Which of the following regarding non-Hodgkin lymphoma (NHL) is correct?
a) Low grade NHL: potentially curable, short survival without treatment
b) Low grade NHL: potentially curable, long survival without treatment
c) High grade NHL: potentially curable, short survival without treatment
d) High grade NHL: not curable, short survival without treatment
e) High grade NHL: not curable, long survival without treatment
24.1) What is the treatment of choice for asymptomatic high grade (Stage III and IV)
non-Hodgkin lymphoma?
a) No treatment exists
b) Observation (initially)
c) Oral chlorambucil daily
d) CVP (cyclophosphamide, vincristine, prednisone)

DO NOT DISTRIBUTE - 25 -
Hematology & Oncology – Part 2 14Mar2009

e) CHOP (cyclophosphamide, doxorubicin, Oncovin, prednisone)


24.2) Which of the following is NOT a treatment choice initially for symptomatic high
grade NHL?
a) Rituximab (anti CD20 antibody)
b) Oral chlorambucil daily
c) CVP (cyclophosphamide, vincristine, prednisone)
d) Fludarabine and interferon therapy
e) CHOP (cyclophosphamide, doxorubicin, Oncovin, prednisone)
25) Which of the following is used for relapse of low grade (Stage I and II) NHL?
a) Rituximab (anti CD20 antibody)
b) Oral chlorambucil daily
c) CVP (cyclophosphamide, vincristine, prednisone)
d) Fludarabine and interferon therapy
e) CHOP (cyclophosphamide, doxorubicin, Oncovin, prednisone)
26) A hospitalized patient is diagnosed with gastric MALT lymphoma and started on
amoxicillin. What bacterial infection is associated with this neoplasm?
a) S. auerus
b) B. cereus
c) C. perfringens
d) H. pylori
e) E. faecalis
27) What is the chemotherapy regimen of choice in the treatment of intermediate grade
NHL or diffuse large cell lymphoma?
a) ABVD (Adriamycin, bleomycin, vinblastine, dacarbazine)
b) BEACOPP (bleomycin, etoposide, Adriamycin, cyclophosphamide, Oncovin,
procarbazine, prednisone)
c) MOPP (Mechlorethamine, Oncovin, procarbazine, prednisone)
d) ChlVPP/EVA (chlorambucil, vincristine, procarbazine, prednisone, etoposide,
vinblastine, Adriamycin)
e) CHOP (cyclophosphamide, doxorubicin, Oncovin, prednisone)
28) What is the treatment option used in the management of intermediate grade and high
grade NHL in sensitive relapse?
a) Radiation therapy
b) ABVD therapy
c) MOPP therapy
d) BEACOPP therapy
e) Bone marrow transplant
29) A patient is found to have monoclonal protein. Bone marrow examination reveals
plasma cells are less than 10% with no evidence of bone lesions. Labs show no anemia,
no hypercalcemia, and no renal insufficiency. Electrophoresis shows mostly IgG. M
protein is < 3g/dL. Monoclonal gammopathy of undetermined significance (MGUS) is
diagnosed. Which of the following might this progress to?
a) AIDS
b) Multiple myeloma
c) Chronic lymphocytic leukemia
d) Hepatitis C

DO NOT DISTRIBUTE - 26 -
Hematology & Oncology – Part 2 14Mar2009

e) Waldenström macroglobulinemia
30) A 65-year-old African American man presents with weakness, fatigue, and bone pain.
Lab tests show anemia, renal insufficiency, and hypercalcemia. Serum and urine are
positive for paraprotein. Radiographs show “punched out” lytic lesions with osteoporosis
and possible fractures. Which of the following is most likely?
a) MGUS
b) Smoldering myeloma (SMM)
c) Multiple myeloma (MM)
d) Plasmacytoma (solitary myeloma)
e) Waldenström macroglobulinemia
31) A patient undergoes testing which finds M protein > 3g/dL and more than 10%
plasma cells in bone marrow. No lytic bone lesions are found. Treatment for this patient
is observation. Which of the following is most likely?
a) MGUS
b) Smoldering myeloma (SMM)
c) Multiple myeloma (MM)
d) Plasmacytoma (solitary myeloma)
e) Waldenström macroglobulinemia
32) What is the recommended treatment for multiple myeloma in patients over age 70?
a) Melphalen and prednisone
b) Lenolinomide
c) Dexamethasone and thalidomide
d) Peripheral stem cell transplant
e) Pallitive radiotherapy
33) What is the initial treatment for multiple myeloma in patients under age 70?
a) Melphalen and prednisone
b) Lenolinomide
c) Dexamethasone and thalidomide
d) Peripheral stem cell transplant
e) Pallitive radiotherapy
34) A patient presents with fatigue, dizziness, blurred vision, and bleeding from the
gums. Retinal venous congestion (“sausage formation”) is found on examination.
Electrophoresis shows IgM paraprotein > 3g/dL as well as Bence-Jones proteinuria.
Which of the following is most likely?
a) MGUS
b) Smoldering myeloma (SMM)
c) Multiple myeloma (MM)
d) Plasmacytoma (solitary myeloma)
e) Waldenström macroglobulinemia
35) What is the treatment for a patient with only an IgM MGUS?
a) Stem cell transplant
b) Observation (initially)
c) Melphalen and prednisone
d) Dexamethasone and thalidomide
e) Palliative radiotherapy

DO NOT DISTRIBUTE - 27 -
Hematology & Oncology – Part 2 14Mar2009

36) Which of the following is the recommended treatment of choice for hyperviscosity
syndrome?
a) Plasmapheresis
b) Chlorambucil
c) Cladribine (2-CDA)
d) Fludarabine
37) Although many therapies exist for Waldenström macroglobulinemia, which of the
following should be used initially in an emergency situation?
a) Plasmapheresis
b) Alkylating agents
c) Interferon alpha
d) Purine neucleoside analogs
e) High-dose chemotherapy
38) What histological staining technique is used to identify amyloidosis?
a) Prealbumin
b) Periodic acid schiff (PAS)
c) Congo red stain
d) Mallory trichrome stain
e) Hematoxylin and eosin (H&E)
39) Which of the following is NOT associated with amyloidosis?
a) Macroglossia
b) Carpal tunnel syndrome
c) Splenomegaly
d) Congestive heart failure
e) Nephrotic syndrome
40) What is the most important screening test used for suspected amyloidosis?
a) CBC with differential
b) Kidney function tests
c) Prealbumin and calcium level
d) Serum or urine M protein
e) Creatinine clearance
41) A patient presents with suspected amyloidosis. Bone marrow shows plasma cells with
clonal predominance of a light chain isotope. A fat biopsy of the abdominal wall is
performed. If an ECG were performed, which of the following is most likely to be seen?
a) No electrocardiographic abnormalities
b) Narrow QRS complexes with increased PR interval
c) Drastically increased QT interval and possible Torsade de Pointes
d) Low voltage pattern consistent with myocardial infarction
e) Diffuse ST elevation and PR depression, mimicking pericarditis

Hematology/Oncology #7 – Clinical: Acute Leukemias


1) A 65-year-old patient presents with bleeding gums and a sore
throat. Blood tests reveal anemia, a high WBC count, and many
WBC precursor “blast” cells. A peripheral blood smear is shown,
which display WBCs with clumps of azurophilic granular material
that form elongated needles. Which of the following is most likely?

DO NOT DISTRIBUTE - 28 -
Hematology & Oncology – Part 2 14Mar2009

a) Acute lymphoblastic leukemia (ALL)


b) Chronic lymphocytic leukemia (CLL)
c) Small lymphocytic leukemia (SLL)
d) Acute myelogenous leukemia (AML)
e) Follicular lymphoma
2) A patient with AML is brought from the oncology department to the emergency
department, suspecting neulogical complications. A cerebral hemorrhage is discovered.
Which of the following therapies would NOT help this patient?
a) Hydration
b) Allopurinol
c) Hydroxyurea
d) Cranial irridation
e) Acidification of urine
3) What is the goal of induction therapy in AML?
a) Achieve remission when there is evidence of leukemia
b) Achieve remission when there is no evidence of leukemia
c) Achieve 50% reduction in the number of leukemia cells
d) Achieve 66% reduction in the number of leukemia cells
e) None of the above
4) What is the pathognomonic finding associated with AML?
a) Risus sardonicus
b) Koplik spots
c) Auer rods
d) Congo red birefringence
e) “Hairy” cells
5) Which of the following is induction chemotherapy for AML
a) Melphalen, lenolinomide, and prednisone
b) Dexamethasone and thalidomide
c) Cytarabine and daunorubicin
d) Stanford V or ABVD
e) MOPP or BEACOPP
6) What is the initial subtype of AML that is most associated with disseminated
intravascular coagulation (DIC) and Auer rods?
a) M0
b) M1
c) M2
d) M3
e) M4
7) Which of the following forms of post-remission therapy is meant to avoid severe bone
marrow suppression?
a) Maintenance therapy
b) Consolidation therapy
c) Intensification therapy
8) Allogeneic bone marrow transplantation for AML patients is recommended two years
after remission. Which of the following patients has the best prognosis?
a) < 30 years old with an HLA match

DO NOT DISTRIBUTE - 29 -
Hematology & Oncology – Part 2 14Mar2009

b) < 20 years old with antecedent hematologic disease


c) > 30 years old and < 55 years old
d) > 55 years old with an HLA match
e) > 55 years old with antecedent hematologic disease
9) Which of the following is the most common antigen in acute lymphocytic
(lymphoblastic) leukemia (ALL)?
a) CD5
b) CD10
c) CD19
d) CD20
e) CD23
10) Which of the following ALL patients has the poorest prognosis?
a) 55 years old with bleeding gums and bone pain
b) 40 years old with WBC > 30,000 and Ph1-positive
c) 70 years old with splenomegaly
d) 18 years old with hepatomegaly, splenomegaly, and lymphadenopathy
e) 5 years old with WBC < 20,000
11) What agents are used for induction chemotherapy in ALL?
a) Stanford V or ABVD
b) Dexamethasone and thalidomide
c) Cytarabine and daunorubicin
d) Melphalen, lenolinomide, and prednisone
e) Vincristine, daunorubicin, prednisone
12) What are the indications for allogeneic bone marrow transplant in ALL?
a) Burkitt t(8;14) and Follicular t(14;18) abnormalities
b) AML M3 t(15;17) and Philadelphia t(9;22) abnormalities
c) Ewing t(11;22) and Mantle t(4;11) abnormalities
d) Philadelphia t(9;22) and Mantle cell t(4;11) abnormalities
e) Burkitt t(8;14) and Ewing t(11;22) abnormalities

Hematology/Oncology #8 – Clinical: Chronic Myeloid Disorders


1) Which of the following is NOT a common clinical presentation of myelodysplastic
syndromes (MDS)?
a) Infection
b) Hemorrhage
c) Anemia
d) Cytopenia
e) DIC
2) What is the most common life-threatening complication seen in MDS, especially in
patients with refractory anemia?
a) Acute lymphoblastic leukemia (ALL)
b) Chronic lymphocytic leukemia (CLL)
c) Acute myelogenous leukemia (AML)
d) Chronic myelogenous leukemia (CML)
e) Disseminated intravascular coagulation (DIC)

DO NOT DISTRIBUTE - 30 -
Hematology & Oncology – Part 2 14Mar2009

3) Which of the following treatment strategies is specific for MDS patients older than 55
years of age?
a) Allogeneic bone marrow transplant
b) Supportive with RBC transfusion, erythropoietin, and antibiotics
c) 5-azacytidine, antithymocyte globulin, and amifostine
d) Antileukemia chemotherapy

Hematology/Oncology #9 – Clinical: Chronic Myelogenous Leukemia


1) What chromosome translocation is the hallmark for nearly all patients with chronic
myelogenous leukemia (CML), leading to a fused bcr-abl oncogene?
a) t(8;14)
b) t(9;22)
c) t(14;18)
d) t(15;17)
e) t(11;22)
f) t(11;14)
2) What is the most common cell type seen in CML?
a) Myeloblasts
b) Megakaryocytes
c) B lymphoblasts
d) Erythroblasts
e) Monoblasts
3) Which of the following, along with older age, is a poor prognostic factor in CML?
a) Low WBC count
b) High WBC count
c) Low platelet count
d) High platelet count
e) Low RBC count
4) A middle-aged man presents with easy bruising and malaise. Physical exam reveals
splenomegaly. Blood tests show increased neutrophils and very low leukocyte alkaline
phosphatase. Which of the following is most likely?
a) Acute lymphoblastic leukemia (ALL)
b) Chronic lymphocytic leukemia (CLL)
c) Acute myelogenous leukemia (AML)
d) Chronic myelogenous leukemia (CML)
e) Hairy cell leukemia
5) Which of the following would NOT present with a very low (or zero) leukocyte
alkaline phosphatase (LAP) score?
a) Paroxysmal nocturnal hemoblobinuria
b) Infectious mononucleosis
c) Aplastic anemia
d) CML
e) HIV
6) A middle-aged man undergoes chromosome analysis, revealing the presence of the
Philadelphia chromosome t(9;22). What is the standard first line therapy for this patient?
a) Vincristine (Oncovin)

DO NOT DISTRIBUTE - 31 -
Hematology & Oncology – Part 2 14Mar2009

b) Vinblastine (Velbe)
c) Imatinib (Gleevec)
d) Paclitaxel (Taxol)
e) Hydroxyurea (Hydrea)
7) What is the treatment of choice for CML with high blast counts and leukostatic
lesions, with safety in thrombocytopenia?
a) Vincristine (Oncovin)
b) Vinblastine (Velbe)
c) Imatinib (Gleevec)
d) Paclitaxel (Taxol)
e) Hydroxyurea (Hydrea)
8) When CML is in the accelerated phase, patients often have fever, poor appetite, weight
loss, and new chromosome changes. Which of the following is diagnostic of the blast
phase (“blast crisis”) in CML?
a) Immature WBCs < 10%
b) Mature WBCs < 10%
c) Immature WBCs > 30%
d) Mature WBCs > 30%
e) Decrease in LAP score

Hematology/Oncology #10 – Clinical: Myeloproliferative Disorders


1) What is the hallmark of agnogenic myeloid metaplasia (AMM, primary idiopathic
myelofibrosis, myelosclerosis), seen in 100% of patients?
a) Dacrocytes (teardrop RBCs)
b) Hypocellular bone marrow
c) Leukoerythroblastic peripheral blood smear
d) Panhyperplasia to osteosclerotic bone marrow
e) Splenomegaly
2) Which of the following is NOT categorized as a myeloproliferative disorder?
a) Chronic granulocytic leukemia
b) Agnogenic myeloid metaplasia
c) Acute myelogenous leukemia
d) Essential thrombocythemia
e) Polycythemia rubra vera
3) Which of the following is a clonal hematologic disorder where patients are
asymptomatic, have a risk of miscarriage or acute leukemic transformation, but have a
near-normal life expectancy?
a) Chronic granulocytic leukemia
b) Agnogenic myeloid metaplasia
c) Acute myelogenous leukemia
d) Essential thrombocythemia
e) Polycythemia rubra vera
4) Which of the following would be seen in a patient with essential thrombocythemia?
a) Philadelphia chromosome translocation
b) Abnormal RBC mass
c) Collagen fibrosis

DO NOT DISTRIBUTE - 32 -
Hematology & Oncology – Part 2 14Mar2009

d) Stainable iron in bone marrow


e) Reactive thrombocytosis
5) Platelet apheresis is the treatment for essential thrombocythemia in all of the following
cases EXCEPT:
a) Acute bleeding
b) Age < 30 years
c) Elective surgery
d) During delivery
e) None of the above
6) What is the treatment of choice for essential thrombocythemia patients with
thrombotic symptoms, cardiovascular risk, or those who are > 60 years old with a platelet
count > 600x10^9/L?
a) Hydroxyurea
b) Anagrelide
c) Anti-aggregating agents
d) Aspirin
e) Dipyridamole
7) What hemoglobin concentration is the cut-off for absolute erythrocytosis in males?
a) 13.5g/dL
b) 15.1g/dL
c) 16.5g/dL
d) 18.0g/dL
e) 21.3g/dL
8) A patient presents with weakness, headache, dyspnea, epigastric distress, and
complaints of pruritus after bathing. Lab tests show increased RBC mass, increased
platelet count, a LAP score > 100, and a high serum vitamin B12 level. Physical exam
finds splenomegaly. History reveals occasional gout. Bone marrow aspiration shows
absent iron stores and tri-lineage hyperplasia. Which of the following is most likely?
a) Chronic granulocytic leukemia
b) Agnogenic myeloid metaplasia
c) Acute myelogenous leukemia
d) Essential thrombocythemia
e) Polycythemia rubra vera
9) What is the mainstay treatment for polycythemia rubra vera (PCV)?
a) Plasmapheresis
b) Platelet apheresis
c) Phlebotomy
d) Hydroxyurea
e) Phosphorus 32

Hematology/Oncology #11 – Oncology: Breast Cancer


1) What is the expected lifetime risk of breast cancer in women?
a) 1 in 4
b) 1 in 8
c) 1 in 12
d) 1 in 16

DO NOT DISTRIBUTE - 33 -
Hematology & Oncology – Part 2 14Mar2009

e) 1 in 20
2) Which of the following is a low risk factor (relative risk 1.0-2.0) for breast cancer in
women?
a) Old age
b) Family history of breast cancer
c) Proliferative disease on breast biopsy
d) Menarche before age 12
e) Homosexuality
3) What is the risk of breast cancer in women with the genes BRCA1 and BRCA2?
a) 20%
b) 40%
c) 60%
d) 80%
e) 100%
4) Annual clinical breast examination and mammography is recommended for all females
older than 50 years. If a patient has risk factors for breast cancer, when should they be
screened?
a) Age 18
b) Age 21
c) Age 30
d) Age 40
e) Age 50
5) A female patient presents to the primary care clinic for a routine examination. Breast
examination reveals a suspicious palpable lump. Mammography is negative. What is the
next course of action?
a) Return for exam in 1 year
b) MRI scan of the breast
c) PET scan of the breast
d) Complete lump excision
e) Lump biopsy
6) What is the recommended treatment for ductal/intraductal carcinoma in situ (DCIS)?
a) Radiotherapy to prevent metastasis
b) Chemotherapy to treat possible metastasis
c) Complete excision including axillary lymph nodes
d) Bilateral mastectomy
e) Local therapy only
7) What is the most common type of invasive breast cancer, seen in 70% of all breast
cancers?
a) In situ breast cancer
b) Invasive ductal carcinoma (IDC)
c) Invasive lobular carcinoma (ILC)
d) inflammatory breast cancer
e) Paget disease of the breast
8) What is the most important prognostic factor for invasive breast cancer?
a) Bilateral involvement
b) Involvement of the superior lateral quadrant (tail)

DO NOT DISTRIBUTE - 34 -
Hematology & Oncology – Part 2 14Mar2009

c) Axillary lymph node involvement


d) Mediastinal/hilar lymph node involvement
e) Tumor size
9) Which of the following patients would have the best prognosis with breast cancer?
a) Testosterone receptor negative
b) Testosterone receptor positive
c) Estrogen receptor negative
d) Estrogen receptor positive
e) Progesterone receptor negative
10) What is the second best indicator for a favorable prognosis in breast cancer?
a) Bilateral involvement
b) Involvement of the superior lateral quadrant (tail)
c) Axillary lymph node involvement
d) Mediastinal/hilar lymph node involvement
e) Tumor size
11) Breast cancers are staged based on tumor size (T1 <2cm, T2 < 5cm, T3 > 5cm, T4 =
extension of chest wall), regional nodes (N1 = ipsilateral axillary, N2 = matted/fixed
nodes, N3 = local metastasis), and metastasis (M1 = distant metastasis). Which of the
following categories would be considered locally advanced disease (non-operable)?
a) T4 N3 M1
b) T2 N1 M0
c) T3 N0 M0
d) T0 N2 M0
e) T2 N2 M1
12) The outcome for women with invasive breast cancer depends on the presence of
distance microscopic metastatic disease, rather than on the treatment of local disease.
How is the presence of micrometastatic disease determined?
a) CT scan measuring tumor size and shape
b) Biopsy and dissection of most of the axillary lymph nodes
c) Sentinel lymph node biopsy using radioactive tracer or blue dye
d) Radionucleotide PET scanning of the breasts
e) Bone marrow biopsy and CSF testing
13) Which of the following is the recommended adjuvant therapy for breast cancer
patients who are estrogen receptor positive?
a) Lumpectomy with axillary lymph node dissection and breast radiation
b) Mastectomy
c) Breast radiation then mastectomy
d) Hormonal therapy
e) Chemotherapy and tamoxifen (Nolvadex)
14) The average survival for patients with recurrent breast cancer is 2.5 years. What is the
initial treatment for advanced breast cancer?
a) Lumpectomy with axillary lymph node dissection and breast radiation
b) Mastectomy
c) Breast radiation then mastectomy
d) Hormonal therapy
e) Chemotherapy and tamoxifen (Nolvadex)

DO NOT DISTRIBUTE - 35 -
Hematology & Oncology – Part 2 14Mar2009

15) The most common combination chemotherapy regimens for breast cancer include all
of the following drugs EXCEPT:
a) Cyclophosphamide
b) Vinblastine
c) Methotrexate
d) Doxorubicin
e) 5-Fluorouracil
16) Which of the following drugs is associated with thromboembolic risk and vaginal
dryness, acts as an anti-estrogen in select tissues, such as the breast, and acts as an
estrogen in other tissues?
a) Tamoxifen
b) Trastuzumab
c) Imatinib
d) Vincristine
e) Palitaxel
17) Aromatase inhibitors (anastrozole, exemestrane, letrozole) increase the risk of:
a) Endometrial cancer
b) Hot flashes and bleeding
c) Thrombocytopenia
d) Osteoporosis
e) Diabetes
18) Aromatase inhibitors are useful in reducing the risk of breast cancer reoccurance,
especially in post-menopausal women, and are beneficial when compared to tamoxifen as
they do NOT have the risk of:
a) Endometrial cancer
b) Hot flashes and bleeding
c) Thrombocytopenia
d) Osteoporosis
e) Diabetes
19) Which of the following drugs, marketed as Herceptin, has activity against HER2/neu,
which is overexpressed in about 25% of breast cancers?
a) Tamoxifen
b) Trastuzumab
c) Imatinib
d) Vincristine
e) Palitaxel
20) Pamidronate (Aredia) can reduce the need for palliative radiation and pain medication
in women with what form of breast cancer metastasis?
a) Lymph
b) Lung
c) Bone
d) Liver
e) Spinal cord
f) Spleen
g) Brain/CNS

DO NOT DISTRIBUTE - 36 -
Hematology & Oncology – Part 2 14Mar2009

Hematology/Oncology #12 – Oncology: Cervical Cancer


1) Which of the following is NOT a risk factor for cervical cancer?
a) First intercourse at an early age
b) Greater number of sexual partners
c) Lower socioeconomic status
d) History of STDs
e) Alcohol use
2) Which of the following is an indication for colposcopy (cervical biopsy)?
a) Women diagnosed with breast cancer
b) Bi-yearly at routine gynecological exams
c) If the cervical os appears inflamed or pyogenic
d) If a Pap smear shows dysplasia or malignancy
e) Women with a family history of endometriosis
3) The American Cancer Society recommends two years of Papanicolaou tests (Pap
smear) for asymptomatic women older than 20 years of age or younger than 20 years of
age if they are sexually active. How often should this test be done after the initial two
years of testing?
a) Every year
b) Every other year
c) Every three years
d) Every five years
e) Screen is no longer needed
4) What is the treatment for early invasive cervical carcinoma as well as the
recommended treatment for carcinoma in situ of the cervix?
a) Total hysterectomy
b) Radiation therapy
c) Combination chemotherapy
d) Cisplatin only
e) No treatment is needed

Hematology/Oncology #13 – Oncology: Colorectal Cancer


1) What type of diet is associated with colorectal cancer?
a) Vegetarian or Mediterranean diet
b) High fiber, high salt diet
c) High fat, low fiber diet
d) High salt, high complex carbohydrates
e) Low simple carbohydrate diet (e.g. Atkins)
2) Regardless of risk, what ages should screening for colorectal cancer begin?
a) Age 18
b) Age 40
c) Age 50
d) Age 65
e) Age 70
3) Which of the following describes the guidelines for colorectal cancer screening in
patients that are at high risk (e.g. family history)?
a) Colon biopsy at regular intervals

DO NOT DISTRIBUTE - 37 -
Hematology & Oncology – Part 2 14Mar2009

b) Colonoscopy at regular intervals


c) Proctoscopy with barium enema every 15 years
d) Esophageal endoscopy every year
e) Abdominal MRI with contract at regular intervals
4) Which of the following patients has the least risk for colon cancer?
a) Patient with inflammatory bowel disease
b) Family history of polyposis syndromes
c) Family history of hereditary non-polyposis colorectal cancer
d) Family history of Lynch syndrome or Gardner syndrome
e) Family history of metastatic prostate cancer
5) What is the preferred curative treatment for carcinoma of the colon or rectum?
a) Intense radiotherapy
b) Chemotherapy with 5-fluorouracil (5-FU)
c) Surgical resection
d) Ultraviolet activated localized radiotherapy
e) Dietary changes
6) Which of the following forms of cancer has the worst prognosis?
a) Rectal
b) Colon
7) Which of the following describes colorectal cancer at AJCC stage II (Dukes stage B)?
a) Submucosal layer, 90% survival
b) Muscularis layer, 90% survival
c) Muscularis layer, 60-80% survival
d) Lymph nodes affected, 60-80% survival
e) Lymph nodes affected, 30-60% survival
8) What is the treatment of choice for stage III colon cancer?
a) Surgical resection
b) 5-fluorouracil (5-FU) and leucovorin (folinic acid)
c) Combination chemotherapy and pelvic irradiation
d) Ultraviolet activated localized radiotherapy
e) No treatment options are available
9) What is the treatment of choice for stage II and III rectal cancer?
a) Surgical resection
b) 5-fluorouracil (5-FU) and leucovorin (folinic acid)
c) Combination chemotherapy and pelvic irradiation
d) Ultraviolet activated localized radiotherapy
e) No treatment options are available
10) What are the biologically targeted agents approved for the treatment of metastatic
colon cancer?
a) Bevacizumab (Avastin) and imatinib (Gleevec)
b) Trastuzumab (Herceptin) and bevacizumab (Avastin)
c) Imatinib (Gleevec) and trastuzumab (Herceptin)
d) Bevacizumab (Avastin) and cetuximab (Erbitux)
e) Imatinib (Gleevec) and cetuximab (Erbitux)
11) Which of the following is NOT recommended for monitoring of patients after
resection of colorectal cancer?

DO NOT DISTRIBUTE - 38 -
Hematology & Oncology – Part 2 14Mar2009

a) Patient history
b) Physical exam
c) Liver function tests
d) Carcinoembryonic antigen (CEA)

Hematology/Oncology #14 – Oncology: Lung Cancer


1) What is the leading cause of cancer mortality in the United States?
a) Colorectal cancer
b) Lymphoma
c) Lung cancer
d) Testicular cancer
e) Cervical cancer
2) Which of the following is NOT true of smoking and lung cancer?
a) Ex-smokers have a lung cancer risk that declines over time
b) 95% of lung cancer in men is due to smoking
c) 80% of lung cancer in women is due to smoking
d) 1-2 packs/day smokers have a 25x greater risk of lung cancer
e) Passive smoking (second hand) does not increase lung cancer risk
3) Which of the following is NOT associated with lung cancer?
a) Radon exposure
b) Smelter workers
c) Alcohol use
d) Iron workers
e) Arsenic exposure
4) A middle-aged man presents with hormonal symptoms. History reveals he has been
smoking 2 packs/day for 25 years (50 pack years). Paraneoplastic syndrome is diagnosed.
X-ray reveals a bulky mediastinal adenopathy. Which of the following is most likely?
a) Small cell carcinoma
b) Large cell carcinoma
c) Squamous cell carcinoma
d) Adenocarcinoma
e) Bronchioloalveolar carcinoma (BAC)
5) Which of the following describes where large cell carcinoma of the lung is usually
located?
a) Lung apex
b) Lung base
c) Centrally
d) Peripherally
e) At the site of infection
6) Which of the following is associated with hypercalcemia due to parathyroid (PTH)
peptide secretion?
a) Small cell carcinoma
b) Large cell carcinoma
c) Squamous cell carcinoma
d) Adenocarcinoma
e) Bronchioloalveolar carcinoma (BAC)

DO NOT DISTRIBUTE - 39 -
Hematology & Oncology – Part 2 14Mar2009

7) Which of the following describes where adenocarcinoma of the lung is usually


located?
a) Lung apex
b) Lung base
c) Centrally
d) Peripherally
e) At the site of infection
8) Which of the following is most associated with non-smokers?
a) Small cell carcinoma
b) Large cell carcinoma
c) Squamous cell carcinoma
d) Adenocarcinoma
e) Bronchioloalveolar carcinoma (BAC)
9) Which of the following is associated with patchy infiltrate and multifocal lesions on
chest radiograph?
a) Small cell carcinoma
b) Large cell carcinoma
c) Squamous cell carcinoma
d) Adenocarcinoma
e) Bronchioloalveolar carcinoma (BAC)
10) What stage of non-small cell lung cancer (NSCLC) is associated with tumor < 2cm
from carina, ipsilateral lymphadenopathy, or invading a resectable structure?
a) Stage I
b) Stage II
c) Stage IIIA
d) Stage IIIB
e) Stage IV
11) What is the treatment of choice for NSCLC at stages I to IIIA?
a) Etoposide (Etopophos)
b) Cisplatin
c) Cyclophosphamide (Cytoxan)
d) Doxorubicin (Adriamycin)
e) Surgery
12) What is the initial treatment of choice for locally advanced non-resectable NSCLC?
a) Surgery
b) Chemotherapy
c) Radiation
d) Chemotherapy and radiation
e) No treatment is available
13) What is the treatment of choice for limited stage small cell lung cancer?
a) Surgical resection
b) Chemotherapy with vincristine (Oncovin)
c) Radiation and surgery
d) Chemotherapy and radiation
e) No treatment is available
14) What is the treatment of choice for extensive stage (IV) small cell lung cancer?

DO NOT DISTRIBUTE - 40 -
Hematology & Oncology – Part 2 14Mar2009

a) Radiation
b) Dual-drug chemotherapy
c) Combination chemotherapy
d) Chemotherapy and radiation
e) No treatment is available

Hematology/Oncology #15 – Oncology: Malignant Melanoma


1) What is the lifetime risk for malignant melanoma in the United States?
a) 1 in 8
b) 1 in 25
c) 1 in 50
d) 1 in 75
e) 1 in 100
2) Which of the following patients is at high risk for developing malignant melanoma?
a) Elderly person who lives near the tropic of Cancer (or Capricorn)
b) Fair-skinned person with many freckles and nevi (moles)
c) Hawaiian surfer who wears SPF-30 lotion on a daily basis
d) Fair-skinned person with several birthmark patches on their body
e) African American female who uses indoor tanning
3) When describing possible melanoma, an ABCD approach is used. A is for asymmetry,
B for border irregularity, C for color variation, and D for diameter (and E for evolving).
What is the minimal diameter in this approach? (about the size of a pencil eraser)
a) 3mm
b) 6mm
c) 9mm
d) 3cm
e) 6cm
4) The Breslow microstaging method is used as the best independent predictor of survival
in malignant melanoma. What does this method measure?
a) Tumor diameter
b) Tumor color variation
c) Tumor speed of evolution
d) Tumor depth
e) Presence of hair protruding from tumor
5) How is malignant melanoma managed?
a) Surgical excision
b) Cryoablation
c) Lymph node dissection
d) Radiation therapy
e) Pallitive treatment
6) What is the treatment of choice in patients with metastatic malignant melanoma, which
accounts for the majority of deaths associated with skin cancer?
a) Surgical excision
b) Cryoablation
c) Lymph node dissection
d) Radiation therapy

DO NOT DISTRIBUTE - 41 -
Hematology & Oncology – Part 2 14Mar2009

e) Pallitive treatment

Hematology/Oncology #16 – Oncology: Ovarian Cancer


1) Although there are no early signs of ovarian cancer, what symptoms might patients
present with?
a) Saddle paresthesia
b) Abnormal menses (amenorrhea)
c) Headaches or nausea
d) Increased libido
e) GI complaints (bloating)
2) What stage of ovarian cancer involves the upper abdomen?
a) Stage I
b) Stage II
c) Stage III
d) Stage IV
3) Which of the following is true for ovarian cancer screening in high-risk women?
a) Pelvic ultrasonography is a very effective method for screening
b) Serum CA 125 is a very effective method for screening
c) Pelvic ultrasonography and serum CA 125 are effective screening methods and
thus should be used for screening high-risk women
d) Pelvic ultrasonography and serum CA 125 are not effective screening methods
and thus should not be used for screening high-risk women
e) Pelvic ultrasonography and serum CA 125 are not effective screening methods
but are reasonable methods anyway for screening high-risk women
4) A patient undergoes a debulking procedure to treat ovarian cancer. However, she
presents again with subsequent ovarian cancer. What is the treatment of choice?
a) Hysterectomy
b) Cryoablation
c) Radiation therapy
d) Chemotherapy with platinum and paclitaxel
e) Chemotherapy with vincristine

Hematology/Oncology #17 – Oncology: Prostate Cancer


1) Risk factors for prostate cancer include high dietary fat, older age, and African
American race. When should prostate specific antigen (PSA) be tested for, according to
the American Cancer Society?
a) At any age, especially if requested by the patient
b) At age > 30
c) At age > 40 without digital rectal examination (DRE)
d) At age > 40 with a digital rectal examination (DRE)
e) At age > 50
2) PSA is most useful in monitoring response to prostate cancer therapy after:
a) Radiation therapy has begun
b) Chemotherapy has begun
c) Prostatic shrinking drugs are given
d) Partial prostatectomy

DO NOT DISTRIBUTE - 42 -
Hematology & Oncology – Part 2 14Mar2009

e) Complete prostatectomy
3) Which of the following therapies is reserved for advanced D2 (M1) prostate disease?
a) Radiotherapy
b) Chemotherapy
c) Androgen deprivation
d) Prostatectomy
e) Vasectomy
4) Which of the following is the preferred method of androgen deprivation as it affects
95% of the androgen production?
a) Orchiectomy
b) Leuprolide
c) Buserelin
d) Goserelin
5) What is the mechanism of action for flutamine, bicalutamide, and nilutamide?
a) LHRH agonists
b) LHRH antagonists
c) Androgen receptor agonists
d) Androgen receptor antagonists
6) Prostate cancer had previously been considered refractory to most chemotherapy
regimens. However, improved response and increased surival have been shown with
regimens containing prednisone and:
a) Vincristine (Oncovin)
b) Vinblastine (Velbe)
c) Imatinib (Gleevec)
d) Paclitaxel (Taxol)
e) Hydroxyurea (Hydrea)

Hematology/Oncology #18 – Oncology: Testicular Cancer


1) Which of the following would NOT increase risk for testicular cancer?
a) Age > 50 years
b) Age > 15 years and < 35 years
c) Cryptorchidism
d) Klinefelter syndrome
2) Testicular cancer types are defined based on:
a) Prostate
b) Vas deferens
c) Seminiferous tubules
d) Cremaster muscle
e) Epididymis
3) Which of the following is NOT used in the evaluation of testicular cancer?
a) Beta human chorionic gonadotropin (BhCG)
b) Alpha fetoprotein (aFP) values
c) Abdominal CT
d) Perineal CT
e) Chest CT
4) What stage of testicular cancer is defined as infradiaphragmatic nodal metastases?

DO NOT DISTRIBUTE - 43 -
Hematology & Oncology – Part 2 14Mar2009

a) Stage 0
b) Stage I
c) Stage II
d) Stage III
5) Which of the following is always true for a pure seminoma?
a) Increased BhCG and increased aFP
b) aFP will not be increased
c) BhCG will not be increased
d) aFP will be increased
e) BhCG will be increased
6) What is the procedure of choice for pathologic diagnosis of testicular cancer?
a) Peripheral blood smear
b) Testicular biopsy
c) Seminal fluid examination
d) Radical orchiectomy
e) Inguinal lymph node biopsy
7.1) What is the treatment of choice for stage I and nonbulky stage II seminomas?
a) Close follow-up
b) Infradiaphragmatic lymphatic irradiation
c) Platinum-based chemotherapy
d) Radical orchiectomy
e) No treatment is needed
7.2) What is the treatment of choice for bulky stage II or III seminomas as well as stage II
or III non-seminomas?
a) Close follow-up
b) Infradiaphragmatic lymphatic irradiation
c) Platinum-based chemotherapy
d) Radical orchiectomy
e) No treatment is needed
8) What is the treatment of choice for stage I non-seminomas?
a) Close follow-up
b) Infradiaphragmatic lymphatic irradiation
c) Platinum-based chemotherapy
d) Radical orchiectomy
e) No treatment is needed

Hematology/Oncology #19 – Oncology: Unknown Primary Lesion


1) Which of the following is NOT true of the evaluation of metastatic carcinoma with an
unknown primary lesion?
a) Women with axillary adenocarcinoma should receive breast cancer treatment
b) Women with peritoneal carcinomatosis should undergo surgical sytoreduction
c) Men with bone metastases should undergo a PSA test of blood and the tumor
d) Fine-needle aspiration is preferred over open biopsy for specimen testing
e) Possible treatable malignancies, such as germ cell tumors, must be ruled out
f) All of the above are true

DO NOT DISTRIBUTE - 44 -
Hematology & Oncology – Part 2 14Mar2009

Hematology/Oncology #20 – Oncology: Paraneoplastic Syndromes


1.1) Which of the following causes hypercalcemia due to PTH-like peptide?
a) Small cell lung cancer
b) Lung cancer (squamous) and breast myeloma
c) Lung, GI, and breast cancer
d) Adenocarcinoma
1.2) Which of the following activates clotting cascade factors leading to venous
thrombosis?
a) Small cell lung cancer
b) Lung cancer (squamous) and breast myeloma
c) Lung, GI, and breast cancer
d) Adenocarcinoma
1.3) Which of the following is associated with Cushing syndrome due to ACTH?
a) Small cell lung cancer
b) Lung cancer (squamous) and breast myeloma
c) Lung, GI, and breast cancer
d) Adenocarcinoma
1.4) Which of the following is associated with peripheral neuropathy due to
autoantibodies?
a) Small cell lung cancer
b) Lung cancer (squamous) and breast myeloma
c) Lung, GI, and breast cancer
d) Adenocarcinoma
2) Common locations for carcinoid syndrome involve the lung, thymus, and ovary. What
is the clinical triad seen in this syndrome?
a) Episodic flushing, diarrhea, and bronchospasm
b) Periodic Raynaud phenomenon, nausea, and vomitting
c) Thunderclap headaches, bleeding gums, and anorexia
d) Weight loss, night sweats, and nausea
e) Muscle weakness, gait disturbance, increased strength with exercise
3) Lambert-Eaton syndrome is associated with small cell lung cancer and presents with
which of the following symptoms?
a) Episodic flushing, diarrhea, and bronchospasm
b) Periodic Raynaud phenomenon, nausea, and vomitting
c) Thunderclap headaches, bleeding gums, and anorexia
d) Muscle weakness, inflammatory myopathy, increased creatine kinase values
e) Muscle weakness, gait disturbance, increased strength with exercise
4) A patient presents with a muscle weakness, heliotrope rash, periorbital edema, and
Gottron papules. Blood test show increased creatine kinase values. Dermatomyositis is
diagnosed. Which of the following forms of cancer is likely present, underlying this
current clinical presentation?
a) Small cell lung cancer
b) Lung cancer (squamous) or breast myeloma
c) Lung, GI, or breast cancer
d) Adenocarcinoma

DO NOT DISTRIBUTE - 45 -
Hematology & Oncology – Part 2 14Mar2009

Hematology/Oncology #21 – Oncology: Chemotherapy


1) Which of the following forms of cancer is NOT sensitive to chemotherapy?
a) Germ cell tumors of the testis
b) Germ cell tumors of the ovary
c) Pancreatic and gastric cancer
d) Breast cancer and ovarian cancer
e) Choriocarcinomas
f) Small cell cancer of the lung

Hematology/Oncology #22 – Oncology: Oncologic Complications & Emergencies


1) Which of the following is true in patients with malignancy?
a) Thyroid hormone increased
b) Thyroid hormone decreased
c) Parathyroid hormone increased
d) Parathyroid hormone decreased
2) What is the electrolyte effect of tumors that secrete parathyroid-related protein?
a) Hypercalcemia
b) Hypocalcemia
c) Hypermagnesemia
d) Hypomagnesemia
e) Hypernatremia
3) Which of the following is NOT a clinical sign of hypercalcemia?
a) Anorexia, nausea, constipation
b) Polyuria, polydipsia, dehydration
c) Apathy, somnolence, coma
d) Hypotension, long QT interval, digitalis resistance
4) Which of the following malignancies would NOT be associated with changes in serum
calcium levels?
a) Burkitt lymphoma
b) Multiple myeloma
c) Hodgkin disease
d) Hypernephroma
e) Adenocarcinoma
5) A patient presents to the Emergency Room with a mental status changes. His friend
says he has been drinking a great deal of water. Blood tests show serum calcium is at
11.8mg/dL and serum albumin is at 2.5g/dL. Given that the conversion factor is
0.8mg/dL per 1.0g/dL of serum albumin, how should total calcium be adjusted?
a) 2.5g/dL * 0.8 + 11.8 = 13.8mg/dL corrected serum calcium
b) (4.0g/dL – 2.5g/dL) * 0.8 + 11.8 = 13mg/dL corrected serum calcium
c) (4.0g/dL + 2.5g/dL) * 0.8 + 11.8 = 17mg/dL corrected serum calcium
d) 2.5g/dL + 0.8 * 11.8 = 11.9mg/dL corrected serum calcium
e) (4.0g/dL – 2.5g/dL) + 0.8 * 10.4 = 13mg/dL corrected serum calcium
6) What is the initial step in the management of acute hypercalcemia?
a) Loop diuretics (e.g. furosemide)
b) Bisphophonates given IV
c) IV normal saline hydration

DO NOT DISTRIBUTE - 46 -
Hematology & Oncology – Part 2 14Mar2009

d) Gallium nitrate or mithramycin given IV


e) Subcutaneous calcitonin
7) Which of the following is most associated with tumor lysis syndrome?
a) Lymphoma/leukemia
b) Solid tumors
c) Prostate tumors
d) Abdominal tumors
e) Cerebral/spinal tumors
8) Which of the following is decreased in tumor lysis syndrome?
a) Uric acid value
b) Potassium value
c) Phosphate value
d) Calcium value
9) Which of the following therapies, in addition to hydration and alkalinization, is
recommended before chemotherapy for prophylaxis against tumor lysis syndrome?
a) Calcitonin
b) Allopurinol
c) Hydroxyurea
d) Indomethacin
e) Fresh frozen plasma
10) Febrile neutropenia is diagnosed with absolute neutrophil count of < 500x10^9/L as
well as a fever that falls under which of the following categories?
a) > 38.5ºC once or > 38.0ºC three times
b) > 39.0ºC once or > 38.5ºC three times
c) > 37ºC for 24-hours
d) > 38ºC for 72-hours
e) > 40ºC at any point
11) What is the treatment of choice for febrile neutropenia?
a) Acetaminophen
b) Cool saline IV-drip
c) IV antivirals
d) Ice-tub cooling
e) Broad spectrum antibiotics
12) A prostate cancer patient develops thoracic pain that radiates around the rib cage as a
bilateral band. The patient says the pain is worse with coughing and sneezing. Bone scan
reveals metastases with epidural compression. What is the initial treatment of choice?
a) Surgical excision
b) Radiation therapy
c) Chemotherapy
d) Dexamethasone
e) Acetaminophen

Hematology/Oncology #23 – Pain Management


1) Which of the following is NOT part of the initial pain assessment?
a) Physical exam emphasizing neurological and musculoskeletal
b) History including pain intensity using a scale such as the Wong-Baker FACES

DO NOT DISTRIBUTE - 47 -
Hematology & Oncology – Part 2 14Mar2009

c) Perform a psychosocial assessment and evaluation


d) Prescribe a short duration supply (5 days) of opiate pain medication
e) Perform an appropriate diagnostic workup to determine the cause of the pain
2) Since all pain is not the same, a patient pain history is important for determining
etiology. Which of the following is NOT a critical element of the pain history for a
patient?
a) Previously attempted treatments
b) The OTC medications that make the pain worse
c) How the pain has developed or evolved
d) The patient’s current level of function and impairment
e) Description of the pain and pattern over time
3) Pain is considered chronic if it persists for more than:
a) 1-week
b) 3- to 6-weeks
c) 2-months
d) 3- to 6-months
e) 6- to 12-months
4) Which of the following describes breakthrough pain?
a) Short periods of analgesia in patients with extreme acute pain, such as seen as a
defense mechanism in trauma patients
b) Short periods of extreme pain in patients with acute pain, such as seen during
activity in patients with sports injuries
c) Short periods of extreme pain in patients with chronic pain, such as seen in
cancer patients
d) Short periods of analgesia in patients with chronic pain, such as seen in
diabetic neuropathy and rheumatoid arthritis
5) Which of the following is considered the most appropriate and most effective scale for
measuring a patient’s pain?
a) Unidimensional pain scale from 1 (little pain) to 10 (extreme pain)
b) Unidimensional pain scale from 1 (little pain) to 5 (extreme pain)
c) Visual analog pain scale from one end (no pain) to the other (extreme pain)
d) Pain faces scale showing 6 to 8 different facial expressions for pain
e) Any of the above as long as it is used consistently
6) How are McGill pain questionnaire, the Memorial Pain Assessment Card, and the brief
pain inventory (BPI) categorized?
a) Multidimensional pain scales
b) Unidimensional pain scales
c) Visual analog pain scales
d) Faces pain scales
e) All of the above
7) Since a patient’s pain is often multidimensional, what is a good method for them to
help keep track and describe the pain to clinicians?
a) Email the clinician when in great pain
b) Visit the primary care clinic when in any pain, for documentation
c) Visit the emergency room when in any pain, for documentation

DO NOT DISTRIBUTE - 48 -
Hematology & Oncology – Part 2 14Mar2009

d) Discuss the pain with a spouse or relative so they can give a new and possibly
better perspective to the clinician
e) Create a personal pain diary
8) Which of the following best describes the major goal(s) of pain management?
a) To relieve pain and suffering
b) To return the patient to their occupation
c) To relieve pain and improve function
d) To extend the patient’s lifetime
e) To improve the patient’s average quality of life (QOL) years
9) Which of the following patients would benefit the most from psychological
intervention in pain management?
a) Adolescents with moderate pain
b) Adults with acute back pain
c) Males with work stress who have bone fractures
d) Young females who are pregnant for the first time
e) Patients with chronic or recurrent pain
10) Physical rehabilitation is most useful in what type of patient?
a) Acute cancer pain
b) Acute non-cancer pain
c) Chronic cancer pain
d) Chronic non-cancer pain
e) It has not been shown to be cost effective or useful in pain management
11.1) Which of the following is NOT considered a non-opioid analgesic, which are used
for mild to moderate pain or in conjunction with opiates for severe pain?
a) NSAIDs
b) Anti-depressants
c) Acetaminophen
d) Salicyclates
11.2) Which of the following is NOT considered a non-steroidal anti-inflammatory drug
(NSAID) as it does not provide sufficient anti-inflammatory action?
a) Acetylsalicylic acid (Aspirin, Econtrin)
b) Acetaminophen (Tylenol, Paracetamol)
c) Naproxen sodium (Aleve, Naprosyn)
d) Ibuprofen (Advil, Motrin)
e) Indometacin (Indocin, Indomethacin)
12) According to the American Pain Society, which of the following patients should use a
non-opiate analgesic as part of their pain regimen?
a) Children with moderate pain
b) Adults with mild to moderate pain
c) Adults with intense pain
d) Elderly with intense pain
e) All patients with pain should use non-opiate analgesics
13) Which of the following effects is seen both in aspirin and acetaminophen?
a) Anti-platelet activity
b) Anti-inflammatory activity
c) Analgesic activity

DO NOT DISTRIBUTE - 49 -
Hematology & Oncology – Part 2 14Mar2009

d) Gastric mucosa damage


e) Liver damage with alcoholism
14) What is the initial drug of choice in the treatment of osteoarthritis?
a) Acetylsalicylic acid (Aspirin, Econtrin)
b) Acetaminophen (Tylenol, Paracetamol)
c) Naproxen sodium (Aleve, Naprosyn)
d) Ibuprofen (Advil, Motrin)
e) Indometacin (Indocin, Indomethacin)
15) What is the maximum dose over 24-hours for acetaminophen?
a) 1,000mg
b) 2,000mg
c) 3,000mg
d) 4,000mg
e) 5,000mg
16) Acetaminophen use should be cautioned in which patient population?
a) Alcoholics
b) Diabetics
c) MI patients
d) Stroke patients
e) Drug addicts
17) Which of the following drugs is most likely to interact with acetaminophen?
a) Rifampin
b) Warfarin
c) Barbiturates
d) Sulfinpyrazone
e) Aspirin
18) Due to the action of the COX1 enzyme, what organ is most at risk in patients who
take non-selective COX blockers?
a) Brain
b) Kidney
c) Platelets
d) Stomach
e) Liver
19) Which of the following is NOT true?
a) Failure of one NSAID is predictive of failure for all NSAIDs
b) Combining NSAIDs provides no known benefit
c) NSAID toxicity is additive
d) All of the above are true
20) COX-2 inhibitors show comparable efficacy to traditional NSAIDs.
a) True
b) False
21) Which of the following COX-2 inhibitors is NOT contraindicated in sulfonamide
allergy, but has been taken off the U.S. market due to cardiovascular complications?
a) Celecoxib (Celebrex)
b) Rofecoxib (Vioxx)
c) Valdecoxib (Bextra)

DO NOT DISTRIBUTE - 50 -
Hematology & Oncology – Part 2 14Mar2009

22) What is the major risk associated with NSAIDs?


a) Hemorrhage
b) Peripheral edema
c) GI erosion
d) Rashes
e) Anaphylaxis
23.1) Which of the following, when combined with NSAIDs, increases the risk of
bleeding and thus should involve monitoring for prothrombin time?
a) Lithium
b) Digoxin
c) Oral anticoagulants
d) Valproate
e) Antihypertensives
23.2) Which of the following can occur when combining NSAIDs with lithium?
a) Increased lithium concentration
b) Decreased lithium concentration
c) Increased NSAID concentration
d) Decreased NSAID concentration
e) Hemorrhagic stroke
23.3) Which of the following can occur when combining NSAIDs with antihypertensive
agents?
a) Increased antihypertensive effect
b) Decreased antihypertensive effect
c) Increased NSAID effect
d) Decreased NSAID effect
23.4) Renal clearance of what drug is inhibited when NSAIDs are used concurrently?
a) Fluorouracil
b) Digoxin
c) Oral contraceptives
d) Adenosine
e) Grapefruit juice
23.5) Valproate oxidation is inhibited, reducing clearance up to 30%, when combined
with which of the following NSAIDs?
a) Acetylsalicylic acid (Aspirin, Econtrin)
b) Acetaminophen (Tylenol, Paracetamol)
c) Naproxen sodium (Aleve, Naprosyn)
d) Ibuprofen (Advil, Motrin)
e) Indometacin (Indocin, Indomethacin)
23.6) Which of the following occurs when phenytoin is combined with either high dose
salicyclates or ibuprofen?
a) Increased phenytoin concentration
b) Decreased phenytoin concentration
c) Increased NSAID concentration
d) Decreased NSAID concentration
e) Disseminated intravascular coagulation (DIC)

DO NOT DISTRIBUTE - 51 -
Hematology & Oncology – Part 2 14Mar2009

23.7) NSAIDs reduce the clearance of which of the following drugs, leading to increased
plasma concentration of that drug?
a) Atropine
b) Antihistamines
c) Oral contraceptives
d) Diazepam
e) Methotrexate
23.8) Hypoglycemia can occur when NSAIDs are combined with:
a) Phenytoin
b) Antiarrhythmics
c) Oral anticoagulants
d) Sulfonylureas
e) Methotrexate
23.9) Which of the following occurs when NSAIDs are combined with aminoglycoside
antibiotics?
a) Enhanced NSAID clearance
b) Inhibited NSAID clearance
c) Enhanced aminoglycoside clearance
d) Inhibited aminoglycoside clearance
e) None of the above
24) Opioid analgesics are very useful for moderate to severe chronic and cancer-related
pain. The most effective opioid analgesics are:
a) Full delta-receptor agonists
b) Full kappa-receptor agonists
c) Full mu-receptor agonists
d) Partial delta- & kappa-receptor agonists
e) Full mu-receptor antagonists
25) Which of the following is true of the recommendation for IV opioid administration?
a) The IV route is nearly always preferred over PO (oral)
b) Transdermal administration is relatively ineffective
c) They can be administered via rectal, intrathecal, and subcutaneous routes
d) Oral and transdermal routes are easy, effective, and relatively inexpensive
e) Oral route is prefered in the postoperative setting or if rapid effect is desired
26.1) A typical 10mg IV dose of morphine sulfated used for severe pain would be
equivalent to what oral dose?
a) 1mg
b) 3mg
c) 10mg
d) 30mg
e) 100mg
26.2) What is the time to peak analgesia for oral opioids?
a) Immediately
b) 4-10 minutes
c) 15-30 minutes
d) 45-60 minutes
e) 60-120 minues

DO NOT DISTRIBUTE - 52 -
Hematology & Oncology – Part 2 14Mar2009

27) What is the prototype opioid drug to which the potency of all other opioid drugs is
compared?
a) Fentanyl (Sublimaze)
b) Codeine sulfate (Codeine)
c) Morphine sulfate (Morphine)
d) Oxycodone (Oxycontin)
e) Diacetylmorphine (Heroin)
28) Which of the following opioid forms is recommended for chronic pain patients?
a) High-dose
b) Low-dose
c) Long-acting
d) Short-acting
29) What is a typical opioid rescue dose for a patient with break-through pain?
a) Full 24-hour dose
b) 65-75% of a 24-hour dose
c) 45-55% of a 24-hour dose
d) 25-35% of a 24-hour dose
e) 5-15% of a 24-hour dose
30.1) Prophylactic treatment should be given for what opioid side-effect, which most
patients do not develop a tolerance to?
a) Respiratory depression
b) Constipation
c) Nausea/Vomiting
d) Somnolence
e) Reye syndrome
30.2) A child presents with lethargy, confusion, history of nightmares, and bouts of
severe vomiting. After a physical exam, Reye syndrome is suspected. This can occur in
children who are given aspirin when they have which of the following viral infections?
a) HHV-1: Herpes simplex virus 1 (HSV-1)
b) HHV-3: Varicella zoster virus (VZV)
c) HHV-4: Epstein-Barr virus (EBV)
d) HHV-5: Cytomegalovirus (CMV)
e) HHV-6/7: Roseolovirus
31.1) Which of the following drugs would be useful as an anti-emetic for patients who
develop opioid nausea due to reduced gastric motility?
a) Prochlorperazine (Compazine)
b) Dronabinol (Marinol)
c) Scopolamine (Hyoscine)
d) Midazolam (Versed)
e) Metoclopramide (Reglan)
31.2) Which of the following drugs would be useful as an anti-emetic for patients who
develop opioid nausea due to chemoreceptor activation of the medulla trigger zone?
a) Prochlorperazine (Compazine)
b) Dronabinol (Marinol)
c) Scopolamine (Hyoscine)
d) Midazolam (Versed)

DO NOT DISTRIBUTE - 53 -
Hematology & Oncology – Part 2 14Mar2009

e) Metoclopramide (Reglan)
32) Patients with impaired renal function who are taking the opioid meperidine
(Demerol) are at increased risk for:
a) Nephrotoxicity
b) Hepatotoxicity
c) Neurotoxicity
d) Ototoxicity
e) Optic neuropathy
33) Increased sedation can occur when opioids are given with any of the following
EXCEPT:
a) Metoclopramide
b) Butyrophenones
c) Phenothiazides
d) Corticosteroids
e) Antihistamines
34) Which of the following is NOT considered an adjuvant analgesic?
a) Anti-depressants
b) Anti-convulsants
c) Local anesthetics
d) Muscle relaxants
e) NSAIDs
35) Gabapentin (Neurontin) is used to treat pain as a better alternative to certain anti-
convulsants. What is the important side-effect seen in elderly patients?
a) Splenomegaly
b) Ataxia
c) Renal insufficiency
d) Increased libido
e) Complete peripheral neuropathy
36) Which of the following is NOT a reason that many terminally ill patients, according
to a 1,000 patient survey, do not request additional pain medications?
a) Fear of addiction
b) Side effects
c) Pain already controlled
d) Unwillingness to take more pills
37) Addiction, a primary chronic neurobiological disease, is most influenced by which of
the following?
a) Genetic factors
b) Psychosocial factors
c) Environmental factors
d) All of the above
e) None of the above
38) A chronic pain patient presents to the Emergency Room for “pain relief.” They were
not given their opioid pain medication at their primary care office due to “clock-
watching” and were flagged as a “drug-seeker.” Which of the following would
distinguish addiction to under-treated pain (pseudoaddiction)?
a) Behaviors becoming more violent with time

DO NOT DISTRIBUTE - 54 -
Hematology & Oncology – Part 2 14Mar2009

b) Behaviors resolving with effective treatment


c) Behaviors involving use of foul language
d) Behaviors resolving spontaneously
e) Behaviors resolving with the use of a placebo
39) A chronic pain patient presents to the Emergency Room with profuse sweating and
heart palpitations. History reveals his son stole his opioid pain medication to share
amongst the son’s friends. The patient uses foul language and demands opioids. Which of
the following is most likely?
a) Addiction
b) Pseudo-addiction
c) Physical dependence
d) Compulsive opioid use
e) Drug tolerance
40) A chronic pain patient presents to the primary care clinic with complaints of pain.
The patient states that the pain has been getting progressively worse, even though the
patient has been following their opioid medication regimen as prescribed. Which of the
following is most likely?
a) Addiction
b) Pseudo-addiction
c) Physical dependence
d) Compulsive opioid use
e) Drug tolerance
41) A patient with chronic low back pain has their opioid medication changed by a
primary care clinician. The clinician tells the patient to bring back the unused medication.
At the next visit, the patient did not bring their medication, even after receiving a phone
call reminder. Another clinician in the office saw the patient for a refill after their
medication was “stolen.” Computer health-system reports show several Emergency
Room visits. Which of the following is most likely?
a) Drug tolerance
b) Compulsive opioid use
c) Physical dependence
d) Pseudo-addiction
e) Drug craving/preoccupation
42) A young man with moderate pain after a soccer injury returns to the primary care
clinic a day after receiving NSAIDs for pain. He reports that the pain is getting worse and
he needs “something with more of a kick.” When asked about a rehabilitation program
for his injury, he is only concerned about pain relief. The patient is given a new
prescription for a more potent NSAID and asked to return. The patient eventually returns
after two-weeks of missed appointments. Computer health-system reports show he visited
several other primary care clinics over the past two weeks. Which of the following is
most likely?
a) Drug tolerance
b) Compulsive opioid use
c) Physical dependence
d) Pseudo-addiction
e) Drug craving/preoccupation

DO NOT DISTRIBUTE - 55 -
Hematology & Oncology – Part 2 14Mar2009

43) What is the most reliable indicator for patient pain assessment?
a) Neurological examination
b) Musculoskeletal examination
c) Patient self-reported pain
d) Pain reported by close relatives
e) Occupational disability (e.g. sick days)

Hematology/Oncology #24 – Extra: Erythema Multiforme Update


1) Which of the following is the correct order for most benign to more severe for the
disorders of erythema multiforme (EM), toxic epidural necrolysis (TEN), and Steven-
Johnson syndrome (SJS)?
a) EM => TEN => SJS
b) EM => SJS => TEN
c) SJS => EM => TEN
d) SJS => TEN = > EM
2) What mortality percentage is associated with TEN?
a) 1-3%
b) 3-5%
c) 5-25%
d) 15-35%
e) 35-55%
3) What is the likely immune hypersensitivity type involved in EM, TEN, and SJS?
a) Type I
b) Type II
c) Type III
d) Type IV
e) Type V
4) Erythema multiforme is a symmetrical skin disease (“target-like lesions”) commonly
affecting children and young adults. What are of the body is usually spared in EM?
a) Genitals
b) Sacrum
c) Face
d) Limbs
e) Trunk
5) Drugs are the common offending agents in SJS and TEN. What is the common
offending agent in EM?
a) Mycoplasma pneumoniae
b) Streptococcus pneumoniae
c) Herpes simplex virus (HSV)
d) Varicella zoster virus (VZV)
e) Epstein-Barr virus (EBV)
6) Pregnancy is a common cause of EM; what is the most common drug cause of EM?
a) Sulfonamides
b) Allopurinol
c) Phenytoin
d) Tetracycline

DO NOT DISTRIBUTE - 56 -
Hematology & Oncology – Part 2 14Mar2009

e) NSAIDs
7) Up to 50% of EM cases may involve mucous membranes. About what percentage of
EM cases re-occur after treatment?
a) 0%
b) 20%
c) 40%
d) 60%
e) 80%
8) Although EM treatment is mainly supportive, which of the following treatment may
shorten the duration of the rash?
a) Oral antihistamines
b) IV antihistamines
c) Topical corticosteroids
d) Systemic corticosteroids
e) Wet compress with bland emollients
9) Which of the following has been shown to prevent subsequent EM outbreaks?
a) Macrolide antibiotics
b) Chlorhexidine rinse
c) Diphenhydramine elixir
d) Viscous lidocaine
e) Oral anti-virals
10) How are SJS and TEN usually distinguished from each other?
a) Nature of lesions
b) Areas involved
c) Lesion size
d) Duration of illness
e) Body surface area affected
11) Toxic epidermal necrolysis (“dusky coalescent lesions”) is usually defined as
covering how much body surface area (BSA)?
a) < 10%
b) > 10%
c) < 30%
d) > 30%
e) < 50%
12) What is the Nikolsky sign?
a) Dusky erythematosus lesions
b) Lesions covering 10-30% BSA
c) Target-like lesions with surrounding edema
d) Lesions that denude with slight pressure
e) Lesions that bleed with slight pressure
13) An adult presents with target-like lesions on their face, arms, and abdomen. Some
lesions appear dusky. Lab tests show electrolyte imbalance. The patient complains of
nausea and vomiting. Which of the following is most likely?
a) EM
b) SJS
c) TEN

DO NOT DISTRIBUTE - 57 -
Hematology & Oncology – Part 2 14Mar2009

d) Staphylococcal scalded skin syndrome (SSSS)


14) Staphylococcal scalded skin syndrome (SSSS) is nearly exclusive to what
population?
a) Elderly
b) Adults
c) African Americans
d) Caucasians
e) Children
15) Which of the following therapies for TEN can quickly reverse necrolysis?
a) Corticosteroids
b) Antibiotics
c) Immune globulins
d) Plasmapheresis
e) Cyclophosphamide and antivirals

James Lamberg

DO NOT DISTRIBUTE - 58 -
Hematology & Oncology – Part 2 14Mar2009

AnswerKey 18) C 9) A 28) A 41) D


Onc #1 19) C 10) D 29) B
1) D 20) B 11) D Onc #7
2.1) B 21) E 12) B Onc #6 1) D
2.2) D 13) B 1) C 2) E
3.1) D Onc #3 14) C 2) A 3) A
3.2) E 1) A 15) C 3) B 4) C
3.3) B 2) C 16) A 4) D 5) C
4.1) E 3) D 17) B 5) E 6) D
4.2) D 4) B 18) E 6) C 7) A
4.3) E 5) D 19) D 7) D 8) A
4.4) A 6) A 20) A 8) D 9) B
4.5) C 7) B 21) C 9) E 10) B
5) B 8) A 22) C 10) B 11) E
6) C 9) C 23) C 11) A 12) D
7) C 10) D 24) E 12) C
8) D 11) E 25) A 13) E Onc #8
9) A 12) D 14) D 1) E
10) A 13) A Onc #5 15) A 2) C
14) D 1) D 16) D 3) B
Onc #2 15) B 2) A 17.1) B
1.1) D 16) A 3) A 17.2) D Onc #9
1.2) D 17) D 4) D 18) C 1) B
2) A 18) B 5) D 19) A 2) A
3) C 19) A 6) C 20) B 3) D
4.1) C 20) A 7) E 21) E 4) D
4.2) D 21) F 8) E 22) A 5) E
4.3) E 22) E 9) E 23) C 6) C
4.4) A 23) B 10) D 24.1) B 7) E
5) E 24) C 11) B 24.2) A 8) C
6.1) B 25) C 12) B 25) A
6.2) B 26) A 13) E 26) D Onc #10
6.3) C 27) C 14) D 27) E 1) E
7) D 28) D 15) D 28) E 2) C
8.1) C 29) B 16) E 29) B 3) D
8.2) A 30) C 17) E 30) C 4) D
8.3) C 18) D 31) B 5) B
9) E Onc #4 19) E 32) A 6) A
10) A 1) C 20) C 33) D 7) D
11) C 2) C 21) D 34) E 8) E
12) D 3) B 22) E 35) B 9) C
13) B 4) B 23) C 36) A
14) E 5) A 24) D 37) A Onc #11
15) D 6) E 25) B 38) C 1) B
16) A 7) B 26) E 39) C 2) D
17) A 8) E 27) B 40) D 3) D

DO NOT DISTRIBUTE - 59 -
Hematology & Oncology – Part 2 14Mar2009

4) D 9) E 1.3) A 22) C 12) D


5) E 10) C 1.4) C 23.1) C 13) C
6) E 11) E 2) A 23.2) A 14) E
7) B 12) B 3) E 23.3) B 15) C
8) C 13) D 4) C 23.4) B
9) D 14) C 23.5) A
10) E Onc #21 23.6) A
11) D Onc #15 1) C 23.7) E
12) C 1) D 23.8) D
13) E 2) B Onc #22 23.9) D
14) D 3) B 1) D 24) C
15) B 4) D 2) A 25) D
16) A 5) A 3) D 26.1) D
17) D 6) E 4) E 26.2) D
18) A 5) B 27) C
19) B Onc #16 6) C 28) C
20) C 1) E 7) A 29) E
2) C 8) D 30.1) B
Onc #12 3) E 9) B 30.2) B
1) E 4) D 10) A 31.1) E
2) D 11) E 31.2) A
3) C Onc #17 12) D 32) C
4) A 1) E 33) D
2) E Onc #23 34) E
Onc #13 3) C 1) D 35) B
1) C 4) A 2) B 36) C
2) C 5) D 3) D 37) D
3) B 6) D 4) C 38) B
4) E 5) E 39) C
5) C Onc #18 6) A 40) E
6) A 1) A 7) E 41) B
7) C 2) C 8) C 42) E
8) B 3) D 9) E 43) C
9) C 4) C 10) D
10) D 5) B 11.1) B Onc #24
11) D 6) D 11.2) B 1) B
7.1) B 12) E 2) D
Onc #14 7.2) C 13) C 3) C
1) C 8) A 14) B 4) E
2) E 15) D 5) C
3) C Onc #19 16) A 6) A
4) A 1) D 17) B 7) B
5) D 18) D 8) D
6) C Onc #20 19) A 9) E
7) D 1.1) B 20) A 10) E
8) D 1.2) D 21) B 11) D

DO NOT DISTRIBUTE - 60 -

You might also like